Сохранен 602
https://2ch.hk/spc/res/685225.html
Домены arhivach.top и arhivach.site временно не функционируют! Используйте домен ARHIVACH.XYZ.
24 декабря Архивач восстановлен после серьёзной аварии. К сожалению, значительная часть сохранённых изображений и видео была потеряна. Подробности случившегося. Мы призываем всех неравнодушных помочь нам с восстановлением утраченного контента!

ТРЕД ТУПЫХ ВОПРОСОВ №155. МЕМНЫЙ ПЛУТОН edition

 Аноним 11/10/21 Пнд 10:00:48 #1 №685225 
cvbRMeY[1].png
4fa66d50b2f228dedefbeb368fb201ea--incorrect-russia[1].jpg
3222751025185[1].jpg
117137946692707665367915453013284858242275o[1].png
ИТТ задаём самые глупые интересующиеся вас вопросы по теме космоса и всего, с ним связанного.
ИТТ тебе ответят. Или нахуй пошлют. Или проигнорят. Или забанят. Или нет.
Прошлый: >>679708 (OP)
Аноним 11/10/21 Пнд 11:39:24 #2 №685237 
Наткнулся на статью об Андорре, которая карликовое государство без выхода к морю, зажатое Францией и Испанией. Андорра эта намного южнее и ближе к экватору, чем 90% РФ, там ещё и морской климат влияет, но тем не менее, там в горах снег. Как так?
Предположим, у на есть некая планета, у которой по экватору расположена горная гряда, полностью и безразрывно опоясывающая планету, при этом эта планета расположено относительно своей солнца на том ж уровне, что и Земля (или даже ближе к Венере). Будут ли снега на такой гряде даже не смотря на то, что это - экватор?
Аноним 11/10/21 Пнд 12:17:06 #3 №685246 
>>685237
Зависит от высоты гор. На Земле температура падает примерно на градус каждые 150 метров. Если на экваторе например +40 на уровне моря, то линия снегов начнется примерно с шести километров.
Аноним 11/10/21 Пнд 14:41:10 #4 №685273 
Я хочу изобрести двигатель который будет придавать заметное ускорение при минимуме потребления. И получить за это Нобелевку.
Аноним 11/10/21 Пнд 14:41:48 #5 №685274 
>>685273
Поэтому прошу помощи. Идея с магнитиками не взлетела
Аноним 11/10/21 Пнд 14:42:45 #6 №685276 
>>685237
Дружище, если в Антарктиде пробурить глубокую скважину в кору, то там температура будет тоже высокая.
Аноним 11/10/21 Пнд 14:49:19 #7 №685277 
>>685273
Чтобы корова давала больше молока и меньше ела, ее нужно больше доить и меньше кормить. Попробуй применить эти селькохозяйственные методы в двигателестроении.
Аноним 11/10/21 Пнд 14:53:24 #8 №685278 
16339356498110 (1)00.jpg
>>685225 (OP)
Аноним 11/10/21 Пнд 14:53:30 #9 №685279 
>>685237
Все настолько плохо? Об этом классе в шестом на географии рассказывают.
Аноним 11/10/21 Пнд 15:06:19 #10 №685281 
>>685277
Дело не в том чтобы меньше/больше, а в оптимизации расхода ресурса.
Аноним 11/10/21 Пнд 15:32:34 #11 №685288 
Когда нас всех засосет в Тёмный Поток в параллельную вселенную, обсуждаем посос сторонников космологического принципа и противников мультиверса.
Аноним 11/10/21 Пнд 15:39:58 #12 №685292 
>>685288
Таблетки прими, шизик ебаный.
Аноним 11/10/21 Пнд 15:42:02 #13 №685295 
>>685292
Кто шиз, ты шиз, епта.

https://www.youtube.com/watch?v=awl0qt6sg9Q

https://www.guide-to-the-universe.com/dark-flow.html
Аноним 11/10/21 Пнд 15:43:14 #14 №685296 
КОСМОС ПРОСТО
АХАХАХАХАХАХХАХАХАХАХАХ
Аноним 11/10/21 Пнд 15:45:20 #15 №685299 
>>685278
Жалко, мы не в ньюсаче. Я бы тебе за безумные умения поставил бы лойс
Аноним 11/10/21 Пнд 15:49:08 #16 №685301 
Подскажите ютуб каналы где запускают самоделки в космос, в домашних условиях. Обычно можно найти, но это 1-2 видоса, не встречал каналов которые ориентированы на это.
(не обязателльно в космос хотя бы в стратосферу)
Аноним 11/10/21 Пнд 15:51:03 #17 №685302 
>>685276
>>685246
Что же это получается, все земная температура поверхности зажата в тонком приповерхностном слое атмосферы в пару км?
Чем же она тогда определяется? И чем определить такие ебические перепады температур?
Аноним 11/10/21 Пнд 15:53:12 #18 №685303 
>>685302
С некоторой глубины (своей для каждой точки на карте) действие Солнца и атмосферы ослабевает настолько, что на первое место выходят эндогенные (внутренние) факторы и происходит разогрев земных недр изнутри, так что температура с глубиной начинает расти.
Аноним 11/10/21 Пнд 16:28:46 #19 №685310 
>>685295
Уебище, Кузнецов в этом видосе как раз и говорил про то, что все это очень сомнительная хуита, приводя научные публикации независимых групп исследователей, которые никакого "темного потока" не увидели нигде. Ты или через жопу слушал, или слышал только то, что тебе хотелось услышать, что удобно для твоего манямирка.
>Термин «тёмный поток» постепенно входит в научный обиход, однако существование тёмного потока пока надёжно не подтверждено независимыми исследованиями[4]. Так, измерения скоростей 715 скоплений галактик каталога ROSAT с использованием 7-летних данных WMAP и улучшенного фильтра, позволяющего точнее отделить эффект Сюняева — Зельдовича от случайных шумов по особенностям спектра, не выявили согласованного движения скоплений[5].
https://ru.wikipedia.org/wiki/%D0%A2%D1%91%D0%BC%D0%BD%D1%8B%D0%B9_%D0%BF%D0%BE%D1%82%D0%BE%D0%BA
Аноним 11/10/21 Пнд 16:36:56 #20 №685313 
>>685310
>возможная причина возникновения тёмного потока — воздействие массы, в настоящее время находящейся вне пределов видимой части Вселенной
Просьба пояснить, они там совсем, блядь, запизделись, и теперь гравитация из-за внешнего горизонта событий прилетает быстрее света? Или же для подверженных этой гравитации галактик видимая вселенная уже включает то, что ещё не долетело до нас, и это такой способ получения (но не передачи, лол) информации из-за горизонта быстрее скорости света?
Аноним 11/10/21 Пнд 17:00:51 #21 №685316 
>>685302
>Чем же она тогда определяется? И чем определить такие ебические перепады температур?
Солнцем, наклоном оси Земли, разностью теплоемкости между океаном и сушей и вызванным всем этим движением воздушных масс
Аноним 11/10/21 Пнд 17:03:36 #22 №685317 
>>685310
Уебище, Кузнецов в этом видосе как раз и говорил что все это в рамках научного спора первооткрывателей с консервативными пидорахами которым ломают привычную картину мира, приводя публикации независимых групп исследователей которые нашли темный поток.

https://www.researchgate.net/publication/268451915_Probing_the_Dark_Flow_signal_in_WMAP_9_yr_and_PLANCK_cosmic_microwave_background_maps
Аноним 11/10/21 Пнд 17:54:52 #23 №685321 
>>685313
Да нет никакого "темного потока", лол блядь. Никто кроме этого Кашлинского не видит его.
>>685317
Чушок, почему кроме твоего Кашлинского никто никакого "темного потока" не видит? Результат Кашлинского не воспроизводится независимыми группами исследователей, и именно об этом Кузнецов сказал в видосе. Как же упорно ты игнорируешь факты, не вписывающиеся в твой чудесатый манямирок, хуйло.
Аноним 11/10/21 Пнд 18:00:48 #24 №685322 
>>685317
https://www.newscientist.com/article/dn23340-blow-for-dark-flow-in-plancks-new-view-of-the-cosmos/
https://arxiv.org/abs/0910.4233
Аноним 11/10/21 Пнд 18:02:24 #25 №685323 
>>685321
>
>
>
> ряяяя не ломайте мой мирок
На помойку истории, сынок.
Аноним 11/10/21 Пнд 18:24:40 #26 №685329 
>>685323
Найс ты порвалась, дырка.
Аноним 11/10/21 Пнд 18:28:20 #27 №685330 
Насколько массивным должен быть камень в космосе, чтобы за него цеплялся кислород и углекислый газ?
Аноним 11/10/21 Пнд 18:55:26 #28 №685334 
1633967725332.png
>>685329
Порвалась дырка твоей матухи когда она попробовала тёмный поток на зуб, дружище.
Аноним 11/10/21 Пнд 19:10:52 #29 №685336 
Хуя вы его приложики конечно, а пруфы хоть будут или это так, пиздежь?
Аноним 11/10/21 Пнд 19:23:47 #30 №685338 
>>685334
Зачем ты закинула своей мамаше-блядине "темный поток" за щеку, дырка? Тебе нравится наблюдать, как ее ебут в рот бешеные негры?
Аноним 11/10/21 Пнд 20:14:44 #31 №685351 
>>685322
Публикация 15 года полностью опровергает потуги консервативных пидорах из твоей ссылки кстати.
>>685321
>никто никакого "темного потока" не видит?
Чушок, ты видос Кузнецова видел хоть? Он там прямым текстом говорит что после того как зашоренные пидорахи попытались заявить на основе своего анализа что потока нет, от них открестился член их же группs, единственный честный ученый открытый новому
Аноним 11/10/21 Пнд 20:23:58 #32 №685355 
Давно не слышно про HALE, а ведь тема будировалась несколько лет назад довольно активно. Оказывается прогресс идёт.
https://eandt.theiet.org/content/articles/2021/10/solar-powered-aircraft-flown-for-nearly-three-weeks-without-landing/
Тупой вопрос - почему на такие хуйни до сих пор не вешают ДЗЗ и связь? На бумаге круто выглядит же. Что мешает?
Аноним 11/10/21 Пнд 22:02:41 #33 №685387 
Можно ли ебать азари без резинки?
Аноним 12/10/21 Втр 00:04:58 #34 №685408 
Спрошу здесь, потому что в разделе наука одни неадекваты сидят малолетние.

Каким образом из многоклеточных существ появились те самые черви и предки рыб? Ведь даже у 5 мм. червяка количество клеток просто колоссально. Каким образом это возникло из существ уровня простейших?
Аноним 12/10/21 Втр 01:08:15 #35 №685414 
>>685408
Очень медленно. От простейших там было огромное количество эволюционных шагов. Сначала одноклеточные сбивались в одну жопу, потом превратились в простых однородных многоклеточных. Потом одна хуйня образовалась, потом другая, третья. Сами клетки усложнялись, отсеиваясь по внешнему воздействию. Конкретно кольчатые червяки возникли в результате кембрийского взрыва, когда по какой-то причине пошла ультрабыстрая эволюция, и сложность в них вовсе не в количестве клеток, а в разнообразии органелл, органов, подсистем, биомолекулярных механизмов, по сравнению с докембрийским планктоном.

>>685387
Вымышленных персонажей можно ебать как хочешь, никаких последствий.
Аноним 12/10/21 Втр 08:18:35 #36 №685424 
>>685351
>Публикация 15 года
Опять с Кашлинским в качестве автора, лол блядь. Чушка, почему кроме твоего Хуишлинского никто никакого потока не видит?
>Он там прямым текстом говорит
Он там прямым текстом говорит, что независимые группы не воспроизводят этот результат и что все Кашлинского тычут ебалом в его же говно, указывают на отсутствие эффекта, а он игнорирует ниудобные факты прям как ты и продолжает носиться со своими маняинтерпретациями кривых данных как с писаной торбой.
>от них открестился член их же группs
Уебище тупорылое, мнение отдельного долбоеба в науке не значит ничего. Имеют вес только результаты исследований, данные. И чтобы уверенно заявлять о каком-то открытии, нужно, чтобы результат воспроизводился другими, независимыми группами исследователей, никак не связанными с изначальным автором(-ами) предполагаемого открытия. Воспроизводимость - один из ключевых элементов научного метода, без нее наука нихуя не может работать (в этом большая проблема современных т.н. социальных наук). Без воспроизводимости любой шизик может хоть вечный двигатель открыть, хоть машину времени, хоть темный поток и бегать потом, орать о том, что "клятая зашоренная официальная новука нипринимаит мае виликае аткрытие ряяя масоны не пущают!" И вот в случае "темного потока" воспроизводимость нулевая. Поэтому ты сосешь собачий хуй как и Кашлинский с его "открытием".
Аноним 12/10/21 Втр 15:51:38 #37 №685505 
Предлагаю такой вариант изучения Юпитера: так как галлилеевские спутники приливнозахвачены, нужно просто присобачить к поверхности Ио, смотрящей на Юпитер, огромную гигантскую крепёжную станцию (в виде зиккурата или пирамиды - не знаю) и от неё протянуть 420000км цепи. Естественно, не из стали, а из углеродных нанотрубок (нитрилуглеродных). Одна сторона у нас крепится на Ио, а другая торона висит прямо на Юпитером, да так, что условно говоря, можно руку в форточку высунуть и потрогать вершину атмосферы Юпитера. Вот на этом конце цепи размещаем научно-наблюдательную станцию, туда же можно турисов возить, и оттуда же можно запускать юпитеропланы для дрейфа и изучения атмосфера Юпитера, так сказать, вплотную.
Как вас идея? В чём подводные камни?
Аноним 12/10/21 Втр 16:07:58 #38 №685509 
>>685505
Шкила, ты хотя бы постарался правильную цифру из википедии скопировать
420 тыс. км это другой показатель
Аноним 12/10/21 Втр 16:10:10 #39 №685510 
>>685330
Тут все зависит где этот камень будет находится.
Если в межзвездном пространстве далеко от ярких звезд, то однотонный булыжник уже способен вокруг себя собирать протоны и их кондиционировать в водород. Не говоря уж об кислороде.
А вот если рядом звезда, то тут зависит как она пичот. Для Солнечной системы: На орбите Нептуна нужен уже булыжник в десяток километров, а вот на орбите Меркурия уже нужен булыжник тяжелее Земли.
Аноним 12/10/21 Втр 16:15:17 #40 №685511 
>>685510
То есть если вояджеру повезет не столкнуться с чем либо, то со временем он может обрасти пылью, стать массивным булыжником, и собрать вокруг себя подобие атмосферы?
Аноним 12/10/21 Втр 16:19:58 #41 №685513 
>>685509
Ладно, ошибся. 350000 км. Но твой доёб к цифре несущественен. По сути есть что сказать?
Аноним 12/10/21 Втр 16:22:48 #42 №685514 
>>685513
Есть. Засунь свои цепочки из нанотрубок в жопу, и читай внимательнее материалы на темы, которые касаешься
Аноним 12/10/21 Втр 16:31:19 #43 №685515 
>>685514
>и читай внимательнее материалы на темы
Ну так
давай свои материалы, умник, блядь
Аноним 12/10/21 Втр 16:31:41 #44 №685518 
>>685511
Да, но на это понабиться миллионы или миллиарды лет. Это при условии, что он не будет со звездами сближаться близко. А если он влетит в молекулярное облако, то и хватить тыщи лет чтоб мутировать в какой-нибудь йобу. Но таких поблизости нет.
И вообще сейчас, как преодолел гелиопаузу, он начал потихоньку вокруг себя собрать атмосферу и обрастать льдом.
Аноним 12/10/21 Втр 16:35:38 #45 №685520 
ow7MT3BvrlE.jpg
>>685518
Отличная идея для сайфая: все планеты солнечной системы на самом деле всего лишь обросшие спутники цивилизации с соседних звёзд, запущенные в планетарный диск Солнца для исследования и случайно ставшие ядрами будущих планет.
Аноним 12/10/21 Втр 16:44:29 #46 №685521 
>>685505
>пердёжную станцию
Пофиксил, не благодари.
Аноним 12/10/21 Втр 16:55:48 #47 №685522 
image.png
>>685520
Аноним 12/10/21 Втр 16:58:35 #48 №685523 
>>685522
Напомните, из какого созвездия прилетела Оумуамуа?
Аноним 12/10/21 Втр 17:09:59 #49 №685524 
>>685523
Лиры
Аноним 12/10/21 Втр 17:21:15 #50 №685526 
>>685523
Киля
Аноним 12/10/21 Втр 17:22:41 #51 №685527 
> он несколько раз циркулировал по Млечному Пути и, таким образом, мог происходить из совершенно другой части галактики
Аноним 12/10/21 Втр 17:24:26 #52 №685528 
>>685523
Сетки
Аноним 12/10/21 Втр 17:39:02 #53 №685531 
Забавно, что выделение газов из него доказать так и не смогли, а любые попытки вынести обсуждение на предмет искусственного происхождения поднимались на смех.
Не удивлюсь, если военные приложили руку, и зонд к нему все таки был отправлен.
Аноним 12/10/21 Втр 18:26:25 #54 №685542 
>>685505
>В чём подводные камни?
В многотысячекилометровом тросе.
Аноним 12/10/21 Втр 21:28:11 #55 №685561 
>>685505
А смысл? Не проще ли изучать Жуп с орбитальной станции, уютно жарясь там от радиации и периодически скидывая зонды на поверхность? Или ты космическую заправку надумал строить, хитрец?
Аноним 12/10/21 Втр 21:40:19 #56 №685567 
>>685542
>В многотысячекилометровом тросе.
Трос не выгоден же - порвётся в одном месте и пиздарики. надобно цепь делать. А лучше цепь карабинов, чтобы всегда можно было по необходимость пару тройку звеньев вставить в цепью
Ну, а кроме этого? Каких-то пол-миллиона километров какой-то там цепи из сверхпрочного полимера - не так уж и сложно. Сложнее, мне кажется, с радиацией и разницами гравитаций, ведь будет момент, когда гравитация Ио уже всё, а Юп ещё не тянет нагло. И в этот моменте будет просадка. На пару км... пару сотен тысяч км.
Вот если сразу все пол-миллиона км запустить - будет попроще.
>>685561
>А смысл?
Ты чё, охуел? Конечно же, ради науки!
Аноним 12/10/21 Втр 22:05:34 #57 №685570 
16252740797610.jpg
Допустим, на орбите Земли есть некий продвинутый враждебный космофлот с большими, но ограниченными ресурсами. Нюки у них тоже есть, но их мало, не хватит на все крупные города, плюс земляне могут часть перехватить.
Как этому флоту нанести нашей планетке максимальный урон минимальными средствами? Закидать астероидами? Жечь повехность гигантским зеркалом? Как-нибудь еще?
Аноним 12/10/21 Втр 22:20:58 #58 №685572 
>>685570
Предложить земным элиткам заманчиво выглядящие модели управления, ведущие к дегенеративному церебральному сортингу, в обмен на обещания вечной жизни после заморозки мозга. Через 200 лет остатки папуасов зачистить.
Аноним 12/10/21 Втр 22:23:33 #59 №685573 
>>685570
Создать евреев и научить вести себя так, как евреи себя и ведут: просачиваться в СМИ и банки, всячески пропагандировать геноцидальные веяния (межрасовые браки, чалдфри, моду на одежду, котоаря аукнется на здоровье много лет спустя, феменизм и т.д.). Когда населения станет совсем мало, можно захватывать планету.
Аноним 12/10/21 Втр 22:26:03 #60 №685575 
>>685570
Никак. Ты ничего не сделаешь цивилизации которая преодолевает такие расстояния и приближается незаметно.

Тут только загвоздка в том, что и им нахуй не сдалась планета с обезьянами при таком развитии. Только в качестве сафари или исследований
Аноним 12/10/21 Втр 22:32:38 #61 №685577 
1634067134945.png
>>685420
Аноним 12/10/21 Втр 22:41:31 #62 №685579 
>>685575
>Тут только загвоздка в том, что и им нахуй не сдалась планета с обезьянами при таком развитии.
Мужики, смотрите, настоящий тупой в треде!
Это обезьяны им нахуй не сдались, а вот целая планета ресурсов, в том числе с залежами металлов и минералов, да и банально с жидкой водой на поверхности - это тебе не хуй с горы.
Аноним 12/10/21 Втр 22:54:14 #63 №685581 
image.png
>>685579
Ты ебанутый? Какая нахуй целая планета ресурсов? Ты же понимаешь что в любом поясе астероидов таких ресурсов будет дохуища?
Я в ахуе.

Ты живешь на силикатном камне, обезьяна, нам твой алюминий нахуй не нужон
Аноним 12/10/21 Втр 23:02:13 #64 №685583 
Нашли инопланетян в общем.

https://phys.org/news/2021-10-strange-radio-emerge-galactic-centre.html
Аноним 12/10/21 Втр 23:16:43 #65 №685586 
>>685583
Надеюсь это азари
Аноним 12/10/21 Втр 23:18:39 #66 №685589 
>>685586
Надеюсь это раса гигантских футанари.
Аноним 12/10/21 Втр 23:30:05 #67 №685592 

В чем вообще чмысл радиоастрономии?
Там >>685583 ссылаются на другой случай произошедший в нулевые года

>обнаружили пять всплесков радиоволн с длиной волны около 1 метра (частота 330 МГц) в течение семи часов с 30 сентября по 1 октября 2002 г.
>пять всплесков имели одинаковую яркость. , каждая из которых длится около 10 минут и происходит каждые 77 минут
>По состоянию на январь 2007 г. других всплесков обнаружено не было.

> источник может быть когерентным или некогерентным

> большинство известных процессов когерентного излучения маловероятно объясняют источник

Т.е. они получают сигналы, которые после прекращаются. Источник назвать не могут. Объяснения уровня бабы сраки с кофейной гущей.

НО. При этом :
>РЯЯЯ НИКАКИХ ИНОПЛОНЕТЯН НЕТ
Аноним 12/10/21 Втр 23:33:19 #68 №685595 
Как после этого вообще можно с гордой лыбой утверждать что инопланетных источников не было найдено за практически столетие и при этом иметь статус ученой степени, если все найденные сигналы, но неопознанные идут с пометкой может карлик, а может не карлик?
Аноним 13/10/21 Срд 08:20:27 #69 №685607 
на всех ли планетах северный полюс "сверху" планеты?
у всех ли планет восход солнца на востоке (справа на карте)?
могут ли быть планеты, где полюса не северный и южный, а восточный и западный или солнечный и теневой?
Аноним 13/10/21 Срд 08:45:09 #70 №685611 
15544826926610.mp4
image.png
image.png
>Как этому флоту нанести нашей планетке максимальный урон минимальными средствами?
Половина населения планеты с элитой моментально перейдёт на сторону интервентов, и инопланетяне, таким образом, смогут изготовить всё необходимое для воплощения своих идей в реальность здесь же на месте.

>Закидать астероидами? Жечь повехность гигантским зеркалом? Как-нибудь еще?
Всё это упирается в энергию. Если у них энергии нюками не хватает даже для уничтожения всех городов, то реальной силы для уничтожения аж целой планеты, на самом деле, у них нет.
Аноним 13/10/21 Срд 08:45:47 #71 №685612 
>>685570
>>685611
Аноним 13/10/21 Срд 09:51:54 #72 №685614 
>>685595
>Как после этого вообще можно с гордой лыбой утверждать что инопланетных источников не было найдено за практически столетие
У слабых умом дурачков есть привычка любые неопознанные объекты и сигналы списывать на всяких рептилоидов. Но что взять со слабых умом? Нет никаких доказательств, что тот или иной неопознанный источник имеет инопланетное происхождение. Поэтому и утверждается, что следов внеземных цивилизаций во Вселенной не было найдено за все время наблюдений. Все эти неопознанные источники с вероятностью 99,999% имеют естественное происхождение и ни с какими инопланетянами не связаны. Тут работает банальная бритва Оккама.
Аноним 13/10/21 Срд 10:14:21 #73 №685617 DELETED
>>685614
>У слабых умом дурачков есть привычка любые неопознанные объекты и сигналы списывать на всяких рептилоидов. Но что взять со слабых умом? Нет никаких доказательств, что тот или иной неопознанный источник имеет инопланетное происхождение. Поэтому и утверждается, что следов внеземных цивилизаций во Вселенной не было найдено за все время наблюдений. Все эти неопознанные источники с вероятностью 99,999% имеют естественное происхождение и ни с какими инопланетянами не связаны. Тут работает банальная бритва Оккама.
Аноним 13/10/21 Срд 10:25:28 #74 №685618 DELETED
Как эта >>685617 болезнь называется?
Аноним 13/10/21 Срд 10:41:32 #75 №685623 DELETED
>>685618
Антисетипетух.
Аноним 13/10/21 Срд 11:07:43 #76 №685634 DELETED
>>685617
Верунок в рептилоидов жидко рвонькнул. Кроме боевых картиночек с антисетипетухом бедняжка ничего не может. Умственная отсталость не в радость.
Аноним 13/10/21 Срд 11:39:57 #77 №685641 
>>685607
Северный полюс по определению сверху, и солнце всегда по определению восходит на востоке. Просто потому что "Север" и "Восток" это условные наименования.
Аноним 13/10/21 Срд 11:42:57 #78 №685642 
>>685607
>солнечный и теневой полюс
А это возможно если если если ось вращения планеты направлена на солнце
Аноним 13/10/21 Срд 11:52:32 #79 №685644 DELETED
>>685618
>>685634
> Как эта болезнь называется?
> Верунок в рептилоидов жидко рвонькнул. Кроме боевых картиночек с антисетипетухом бедняжка ничего не может. Умственная отсталость не в радость.
Аноним 13/10/21 Срд 12:08:38 #80 №685648 
>>685642
>ось вращения планеты направлена на солнце
Охуительная, блядь, история. И каким же образом эта ось сможет быть направлена на центр системы чаще, чем 2 раза в год? Веретенообразная планета в приливном захвате?
Аноним 13/10/21 Срд 12:22:27 #81 №685649 
>>685281
Инфракрасный двигатель на антиматерии. Огромная вогнутая плита из графена нагревается до 3К прокачиванием теплоносителя, который обрабатывается позитронами в нагревателе (и электронами для баланса). Эффективнее уже ничего не будет
Аноним 13/10/21 Срд 13:34:28 #82 №685660 DELETED
>>685634
Не позорься идиот. боевые картиночки не моих рук дело.
Мой посыл был в том, что кукареки с фразами:

>ну мы нихуя не знаем, может это, а может то, а вообще отъебитесь это же не инопланетяне))0

Это именно кукареки основанные на вере, а не на фактах. И эти люди якобы двигают "науку", где 30% это ДОГАДКИ. Т.е. ничем не отличимо от религии.

Слепая вера в собственную правоту.
Аноним 13/10/21 Срд 13:45:55 #83 №685662 
1558291916369.jpg
Помню какому-то идиоту тут пояснял, что экстремофилы могут существовать в самой разной жопе, но это не значит, что в каждой жопе могут развиться экстремофилы, потому что они заселяют нишу в жопе только после того как образовались в тепличных условиях в другом месте. А сейчас какой-то хуй о моей идее статьи пишет
https://arxiv.org/abs/2110.06144
Аноним 13/10/21 Срд 13:47:30 #84 №685663 
>>685662
Каждый второй в детстве изобретал велосипед
Аноним 13/10/21 Срд 14:23:43 #85 №685666 
Где-то видел утверждение, что ан поверхности Сатурна гравитация 1g, как на Земле, что сели обтянуть Сатурн корочкой, то по всей его поверхность будет этот самый 1g. Как это? Сатурн это же огромная, гигантская планетища, там и гравитация обязана быть гигантской. Я, значит,не понимаю, разницу между весом, гравитацией, тяготением и причём тут g. Объясните.
Это что, получается, мы можем условно говоря, получить 1g н а любом крупном теле, если обтянем его твёрдой поверхностью? То есть, и Уран, и Нептун, и Юпитер, и Солнце (жару и радиацию опустим) и даже какой-нибудь горизонт событий какой-нибудь сверхмассивной чёрной дыры можно обстроить и жить по-человечески с 1g, привычным и родным? При условии, что у нас ДОХУИЩА материала, конечно.
Аноним 13/10/21 Срд 14:26:06 #86 №685668 
https://youtu.be/qhebh9X1tw4
Аноним 13/10/21 Срд 14:28:14 #87 №685669 
image.png
>>685666
>поверхности Сатурна
>как на Земле
>обтянуть Сатурн корочкой
>гигантская планетища
>обтянем его твёрдой поверхностью
>жару и радиацию опустим
>обстроить и жить по-человечески с 1g
Аноним 13/10/21 Срд 14:53:42 #88 №685674 
>>685669
Сатурн же газовый гигант, в его случае поверхностью планеты считается поверхность видимого слоя атмосферы
Аноним 13/10/21 Срд 18:45:49 #89 №685700 
>>685607
Справа-слева, северный-южный. Это неправильно поставленный вопрос. Правильно поставленный:
>у всех ли планет ось вращения перпендикулярна эклиптике?
>у всех ли направление вращения относительно эклиптики совпадает с земным?
Ответ - не у всех. Венера крутится в обратном направлении, а у Урана вообще ось вращения положена "набок", вдоль эклиптики.
Аноним 13/10/21 Срд 19:09:59 #90 №685704 
>>685674
Нет там поверхности, в физическом смысле. Поверхность это граница фазового перехода, например земля-воздух.
На газовых гигантах есть специальные координатные системы, которые привязаны к их гравипотенциалу, атмосферному давлению в 1 земную атмосферу, магнитному полю, оптической видимости планеты в телескоп, и т.п.

Можно считать неким средним уровнем давление в 1 земную атмосферу, например, часто так делают. Причём поскольку Сатурн постоянно перемешивается - сложно даже точно определить его истинную скорость вращения. Лишь недавно посчитали "гравитационно усреднённую" скорость, на основе наблюдений за траекторией зонда Cassini, и расчёт там был пиздец сложный.

>>685666
В том и дело, что он гигантский и не очень плотный, поскольку в основном состоит из газа. Сила притяжения зависит от дистанции до объекта. Размер у Сатурна большой, поэтому дальние частички тянут слабее, и в результате твой вес там будет не очень большим. Если ту же Землю сжать в два раза, то тянуть на поверхности будет в четыре раза сильнее, и человек будет весить не 80кг скажем, а 320 - хотя масса планеты не изменилась. Если всю массу Сатурна ужать до размеров Земли, то на такой "поверхности" тебя нахуй расплющит сразу же.

В природе дохуя объектов, тянущих гораздо сильнее, те же нейтронные звёзды имеют диаметр всего лишь в несколько сраных километров, а тянут намного сильнее любой звезды, настолько что там даже атомы разлагаются нахуй. Именно потому что они маленькие, а масса при этом как у звезды.
Аноним 13/10/21 Срд 21:14:16 #91 №685730 
>>685662
>>685663
>>685666
>>685668
>>685669
>>685674
>>685700
>>685704
Кто из вас миша?
Аноним 13/10/21 Срд 21:32:29 #92 №685741 
>>685278
Жму руку
Аноним 13/10/21 Срд 21:39:21 #93 №685745 
Предлагаю идею получше.
Сооружаем гигантский пылесос. Шланг делаем из нанотрубок.
Запускаем на орбиту Сатурна.
Выкидываем шланг в атмосферу.
Сосем.
Собираем в контейнеры сжиженный газ, транспортируем топливо на Землю.
Бесплатный бесконечный источник энергии готов
Аноним 13/10/21 Срд 21:40:28 #94 №685746 
>>685745
*Естественно бесплатный с тем учетом, что проект будут спонсировать все страны, и делить полученное топливо пропорционально взносам.
Аноним 13/10/21 Срд 21:41:26 #95 №685747 
Помимо этого на луне можно соорудить гигантское хранилище топлива. И завод по производству транспортировочных модулей, чтобы все осуществлялось в автоматическом режиме стабильно в течении года.
Аноним 13/10/21 Срд 21:44:09 #96 №685748 
А лучше всего (и ближе) добывать ресурсы из Солнца. Лететь естественно ночью, чтобы корабли не сгорели.
Аноним 13/10/21 Срд 22:11:44 #97 №685755 
Янус и Япет постоянно совершают орбитальную рокировку. А меняют ли они своё положение в приливном захвате при этом?
Например, оба они имеют вид поверхности как у Земли. И вот сейчас, за час до рокировки, каждый из них приливнозахвачен так, что смотрит в сторону Юпитера своими Африками. Что с ними случится после рокировки? Они останутся смотреть Африками или сдвинутся лицом в направление Азии/Атлантического океана?

Тут зависит высота орбиты и масса тела или нет?

А если бы на орбите Юпитера были бы два Марса, они бы друг друга при рокеровке провернули бы или нет?

И вообще, почему нет газовых гигантов с железокаменными спутниками как Марс, а только лёдосиликатные, как Ганимед?
Аноним 13/10/21 Срд 23:22:06 #98 №685761 
>>685730
Игорь, ты?..
Аноним 14/10/21 Чтв 08:27:00 #99 №685793 
>>685755
>почему нет газовых гигантов с железокаменными спутниками как Марс
Далеко от солнца, железнокаменая начинка разбавляется льдом при формировании

По поводу проворота, думаю нет, скорее они как точки взаимодействуют. Два марса так не смогут, совершенно другие размеры и массы
Аноним 14/10/21 Чтв 11:29:07 #100 №685807 
>>685793
>Два марса так не смогут, совершенно другие размеры и массы
А почему нет? Вроде же наоборот, больше масса, больше энергии кинетической, мощнее прыгают в рокировку, сильнее проворачиваются.
Аноним 14/10/21 Чтв 11:30:33 #101 №685808 
>>685583
>из центра галактики, в котором по нынешнему мнению науки находится квазар, приходит радиосигнал
Нихуясе, в рот мне ноги.
Аноним 14/10/21 Чтв 11:37:58 #102 №685811 
>>685641
Полюса не условны, это же магнит.
Аноним 14/10/21 Чтв 11:53:11 #103 №685812 
>>685811
Полюса (магнитные или те через которые проходят ось вращения) реальны, но определение северности или южности условны, просто исторически сложилось что тот полюс что направлен на Полярную звезду называется северным.
Аноним 14/10/21 Чтв 12:25:33 #104 №685816 
>>685807
Ну хз, попробуй в Universe sandbox 2 заставить Марсы прыгать
Аноним 14/10/21 Чтв 13:17:36 #105 №685822 
>>685808
Ты дурачок или прикидываешься? Такие типы сигналов непостоянны и пропадают/меняются.
Аноним 14/10/21 Чтв 13:38:28 #106 №685826 
>>685822
Какие такие? От квазаров? Так вон пропадали же, менялись.
Аноним 14/10/21 Чтв 14:53:13 #107 №685850 
>>685826
А блять, точно, десятки ученых скромно сказали "не можем назвать источник"
кукаретик с двачей с пеной у рта пишет про квазары
Аноним 14/10/21 Чтв 14:55:58 #108 №685851 
>>685850
Кукаретик с двачей, вытри "пену" со рта.
Мочёные и не такое скажут, лишь бы дали пару чемоданов долларов на исследования. По сути тебе возразить как всегда нечем.
Аноним 14/10/21 Чтв 15:09:26 #109 №685855 
157649904319821707.jpg
5687132894.jpg
CassinipeersoverTitansharzyatmospharetoSaturnssouthpole.jpg
Huygenssurfacecolorsr.jpg
В прошлый раз я хотел ПРЕВРАТИТЬ ЮПИТЕР В КОРИЧНЕВОГО КАРЛАНА.
Но это - в прошлом.
Теперь я хочу ТЕРРАФОРМИРОВАТЬ ТИТАН.
Что мне для этого нужно?
Я слышал, что какие-то водоросли могут ПЕРЕРАБОТАТЬ АЗОТ В КИСЛОРОД, но на Титане нет океана из воды.
Может по нему сначала БАХНУТЬ? Какой мощности нужен ядерный заряд, что бы на Титане появилась ЖИЖА?
Составляем план, короче.
Аноним 14/10/21 Чтв 15:10:33 #110 №685856 
>>685278
ALLO, PLUTON, ETO TI?
TEBYA PLOHO VIDNO
Аноним 14/10/21 Чтв 16:27:22 #111 №685876 
>>685855
>Я слышал, что какие-то водоросли могут ПЕРЕРАБОТАТЬ АЗОТ В КИСЛОРОД
Водоросль-токамак - поистине чудо природы - открыта в 1967 году советским учёным Х. Я. Ебудаловым.
Аноним 14/10/21 Чтв 17:30:08 #112 №685883 
>>685855
Тепло. Много тепла.
И постоянного.
Короче, или тащи к Титану маленькое солнце, или жди пока Солнце опухнет.
Аноним 14/10/21 Чтв 17:31:12 #113 №685884 
>>685851
Ебать ты даун.
Аноним 14/10/21 Чтв 17:36:12 #114 №685887 
>>685884
>пук
Аноним 14/10/21 Чтв 17:48:59 #115 №685889 
fc2RlbGFub3VuYXMucnUvdXBsb2Fkcy8zLzUvMzU0MTU3NzU0OTQ0NV9vcm[...].jpeg
CHERNB9-900x610.jpg
>>685883
>Тепло. Много тепла.
Насколько много то?
Аноним 14/10/21 Чтв 19:34:16 #116 №685902 
8os-large.jpg
139469295634.jpg
Анон, смотри какую шизу придумал. У нас же при исследовании внешней солнечной системы связь хуевая? А тем более со всякими Пионерами, и прочим.

А почему нельзя ебануть в точку Лагранжа между, например, Марсом и Юпитером ретранслятор-репитер с двумя узконаправленными антеннами? Тогда можно будет качать с зонда ганимедские мокрые писечки в 4к, а не ждать пока он три шакальные картинки за сто оборотов передаст.
Аноним 14/10/21 Чтв 22:22:55 #117 №685941 
>>685902
Толку для аппаратов с технологиями прошлого века?
Бюджет не резиновый
Аноним 14/10/21 Чтв 22:34:59 #118 №685944 
Можно ли в теории воспламенить газовый гигант? Что будет с Сатурном, Юпитером?
Аноним 14/10/21 Чтв 22:38:37 #119 №685945 
>>685944
Можно. Надо всего лишь сказать, что газонюх это лох ипаный, который никогда не станет звездой.
Аноним 14/10/21 Чтв 22:47:46 #120 №685948 
>>685902
>А почему нельзя ебануть в точку Лагранжа между, например, Марсом и Юпитером
Потому что такой точки не существует. Марс и Юпитер движутся по орбитам независимо и могут, например, вообще находиться по разные стороны Солнца.
Аноним 14/10/21 Чтв 22:50:39 #121 №685951 
По невыясненным причинам термосфера Нептуна аномально горячая: около 750 К. Для столь высокой температуры планета слишком далека от Солнца, чтобы оно могло так разогреть термосферу ультрафиолетовым излучением.
Аноним 14/10/21 Чтв 23:06:12 #122 №685953 
Может ли крупная (от Марса до Урана) планета-сирота, влетевшая в сферу влияния звезды, остаться под крылом у этой звезды, а не мимопролететь, упасть на звезду, или с размаху врезаться в какиую-нибудь мимопланету?
Будет ли такая орбита округлой, подобной орбитам Венеры и Земли, или она обязательно будет всрато-вытянутой, как у Плутона?
>>685951
Юпитер тоже отдаёт тепла больше, чем поглощает.
Аноним 14/10/21 Чтв 23:20:02 #123 №685954 
Насколько морально и этично отправлять пожизненно заключенных - облагораживать Марс или Луну?
Аноним 14/10/21 Чтв 23:23:06 #124 №685956 
>>685954
Ну и например как бонус проводить лотерею (после 10-15 лет работы), когда несколько из сотен могут получить помилование, вернуться на Землю и попробовать влиться в общество
Аноним 14/10/21 Чтв 23:37:51 #125 №685957 
>>685954
Многие осужденные наверняка полетели бы добровольцами. Но ты доверишь зеку такое сложное и хрупкое оборудование? Потом каково это, если первопроходцами космоса станут зеки в красных робах? Как тогда пиарить космос и полеты? Это мы еще не коснулись морально-этической стороны вопроса. Не, сейчас не 18 век, каторжники в космосе не нужны, тем более что желающих и так будет предостаточно.
Аноним 14/10/21 Чтв 23:43:23 #126 №685958 
>>685957
Им никто и не будет доверять хрупкую работу. А еще в каждом будут вшиты микрокапсулы с ядом на случай бунта.
Аноним 14/10/21 Чтв 23:45:04 #127 №685959 
>>685957
А еще естественно отбирать только психически устойчивых, исключая психопатов. Зеки как раз будут должны подготовить плацдарм, базу, для работы ученых в будущем
Аноним 15/10/21 Птн 00:08:52 #128 №685964 
>>685953
>Может ли
Да.

>Будет ли такая орбита округлой
Если есть другие планеты, то со временем может стать круглой.
Аноним 15/10/21 Птн 00:11:07 #129 №685965 
Может ли теоретически существовать звездная система где орбиты планет очень близко находятся к друг другу? Например с расстоянием чуть больше чем от Земли до Луны
Аноним 15/10/21 Птн 00:22:55 #130 №685967 
>>685965
Могут, но такие системы не устойчивые и очень быстро(по астрономическим меркам) будут скакать по орбитам, когда не устаканятся по взаимным резонансам.
Аноним 15/10/21 Птн 00:23:58 #131 №685968 
>>685965
Как минимум, есть звёздные системы, где газовые гиганты размером с Юпитер находятся на орбите Меркурия.
Аноним 15/10/21 Птн 00:26:29 #132 №685971 
>>685967
>>685968
Мне просто интересно насколько реалистична идея, что такое случится с планетами земной группы, где успеет появится цивилизация. У таких существ будет больше возможностей чем у нас для освоения остальных планет и прогрессировать, по идее, они должны быстрее
Аноним 15/10/21 Птн 00:27:50 #133 №685972 
>>685953
>Может ли крупная (от Марса до Урана) планета-сирота, влетевшая в сферу влияния звезды, остаться под крылом у этой звезды, а не мимопролететь, упасть на звезду, или с размаху врезаться в какиую-нибудь мимопланету?
Могут, но для того чтобы закрепить орбиту этой планете нужно сбросить момент импульса относительно звезды. Тут два варианта, либо планета сирота обладает массивным спутником и вышвыривают его от себя, либо планета обменивается моментом с другими планетами звезды, пидорася их орбиты.
>Будет ли такая орбита округлой, подобной орбитам Венеры и Земли.
Может, но это слишком маловероятно. Нужно чтоб планета влетела под определенной углом с определенной скоростью.
Аноним 15/10/21 Птн 02:17:14 #134 №685983 
Насколько точно вы сможете меня сдеанонить, если я скажу что сейчас:

-ночь
-относительно ясное небо
-нет луны

?
Аноним 15/10/21 Птн 02:19:26 #135 №685985 
SPKoroev.jpg
>>685957
>каково это, если первопроходцами космоса станут зеки в красных робах
Не понял, а в чём проблема-то?
Аноним 15/10/21 Птн 02:21:37 #136 №685987 
>>685983
Абсолютно точно. Ты — хуй.
Аноним 15/10/21 Птн 03:21:31 #137 №685996 
>>685983
>ясное небо
>-нет луны
Меркурий.
Аноним 15/10/21 Птн 08:46:37 #138 №686008 
Читаю сейчас статью на посмотре.ли "Чужое небо", а там
>Другой вариант — резонанс 3:2, как у Меркурия. Тогда солнце будет описывать по небу петли, а не ходить по прямой.
Что-то не могу представить, как оно всё - ну, движение солнца - будет выглядеть для наблюдателя на Меркурии. Нарисуйте, пож, график или рисунок движения светила солнца по меркурианскому небосклону для тупых
>>685971
>прогрессировать, по идее, они должны быстрее
Почему? Замени Луну второй Землёй: что изменится в прогрессе? Да ничего. Первые полёты раньше 1961 года только до орбиты Земли и не более. Всякие викинги, шумеры,китайцы и ацтеки вели бы себя точно так же. И если мы не берём концепт богов/инопланетян то будет пособничества в ускорении научного процесса близостью другой массивной планеты будет около 0
Аноним 15/10/21 Птн 09:52:44 #139 №686016 
>>686008
>Всякие викинги, шумеры,китайцы и ацтеки вели бы себя точно так же
Нет, при такой близости жители обеих планет уже давно бы друг друга заметили, раньше бы выкинули к хуям геоцентризм и попытались в контакт (выложили бы окромное слово ХУЙ из камушков и начали бы искать ответ в телескоп).
Аноним 15/10/21 Птн 10:06:41 #140 №686019 
>>686008
>Нарисуйте, пож, график или рисунок движения светила солнца по меркурианскому небосклону
Поставь себе Celestia, ускорь там время и наблюдай воочию.

>>686008
>Всякие викинги, шумеры,китайцы и ацтеки вели бы себя точно так же.
Интересно, если бы две такие близкие планеты были населены примитивными народами, как быстро они обнаружили бы друг друга и установили контакт? Теоретически, уже обладая примитивной подзорной трубой можно увидеть пламя большого костра. Крупные города ночью можно вообще увидеть невооруженным взглядом.
Аноним 15/10/21 Птн 10:25:52 #141 №686025 
>>686016
Ну ты и тупой.
>выложили бы окромное слово ХУЙ из камушков и начали бы искать ответ в телескоп
На Луне попробуй выложить камушками слово ХУЙ, чтобы его было видно с Земли. Даже с учётом СОВРЕМЕННЫХ ВЫСОКОТЕХНОЛОГИЧНЫХ ЭЛЕКТРОННЫХ ТЕЛЕСКОПОВ, чтобы с поверхности Земли твой мелкий хуй на Луне можно было рассмотреть. Мы уже кидаем взгляды телескопов вглубь космоса, но до сих пор не можем получить хайрез фото поверхности ближайшей к нам планеты - нашей земной Луны - в ультраХД и 4к. Не то, чтобы поверхность Луны увидеть как поверхность Земли, которую ты видишь идя по улице с высоты своего роста, а хотя бы в масштабах пары квадратных метров на площадь экрана монитора всё ещё невозможно.
А теперь представь себе средневековье... вот я даже не будут тебе пояснять, что чтобы пойти на взаимный контакт нужно чтобы обе планеты были равнопригодны и населены людьми с одинаковым уровне технологий (то есть одна планета может просто пустой); или что если бы одна планета была условно развитой до уровня городов а-ля Земля 70х, то всё равно с поверхности другой планеты она была бы просто камнем с тучами и, возможно, если вдруг особо освещённые по ночам города было бы видно с поверхности другой планеты, средневековому жителю проще понять, что это лава и вулканы, а значит планета неблагоприятна ...короче, у нас две планеты со средневековье, предположим, ну так а с какого хуя человеку, которому грубо говоря нужно натаскать дров, засеять поле, отбиваясь плугом от волков, должно ебать желание лезть вверх на Луну с горящим желанием посмотреть ЧЁ ТАМ У ИНОПЛАНЕТЯН. У просто человека эпохи агропанка что на небе Луна, что яма говна - обыденные повседневные явления, не влияющие существенно на его жизнь. Хотя нет, говно можно использовать на удобрения, а луну - нет.
И тем более, ты вообще охуевшая сутулая собака, раз считаешь, что все жители ОБЯЗАНЫ разом отбросить все мысли, идеи и интересы, к которым их приведут их жизни и цивилизации, а вместо этого засунут их в жопу и начнут думать как нам обустроить двач миссию до Луны.
Так и представляю: средневековые века на обоих твоих планетах. Сидят такие Родрик и Годрик на стоге соломы и обсуждают Srednevekovye Space Program - сколько короьвего пердежа нужно собрать в кишки козы, чтобы можно было взлететь вверх до небесной тверди, а потом сколько дров понадобится для сжигания и перелёт на воздушном шаре (который ещё не изобрели, но фантасты, которых сжигали на кострах за ересь, о них рассказывали) до соседней луны, и Родрик такой говорит:
- ...и вот здесь нам понадобится 500 тысяч стогов хвороста, чтобы прилететь на Луну.
А Годрик даёт ему пощёчину и такой:
- Заткнись нахуй, у нас урожай не собран, надо собрать посевы до дождей иначе зимой от голода сдохнем!
Аноним 15/10/21 Птн 10:33:21 #142 №686027 
>Есть ещё один немаловажный закон природы: суммарная масса спутников гиганта в 4000-10000 раз меньше массы самого гиганта. Поэтому, в случае массивного спутника, с массой, сравнимой с земной, масса гиганта должна быть, минимум, в 10 раз больше Юпитера и большой спутник будет один (остальные — астероидоподобная «мелюзга», как в случае Нептуна). Если есть несколько таких, центральная масса будет соответствовать уже коричневому карлику.
Почему? Почему только ОДИН?
Аноним 15/10/21 Птн 10:51:33 #143 №686029 
>>686025
>ЭЛЕКТРОННЫХ ТЕЛЕСКОПОВ
Это как электронный микросокоп? То есть изображение формируется пучком электронов и прочее?
>но до сих пор не можем получить хайрез фото поверхности ближайшей к нам планеты - нашей земной Луны - в ультраХД и 4к.
Для такого достаточно относительно дешевого любительского телескопа, если ты про то что 4к пикселов будет диаметр Луны.
Аноним 15/10/21 Птн 10:57:46 #144 №686031 
>>686025
>импукинг вот этой всей рутины не было во времена открытия америки и прочего поднятия целины. И тем не менее даже за тысячелетия до этого находились персонажи, изучающие движение звёзд и планет.
Аноним 15/10/21 Птн 12:02:43 #145 №686036 
>>686008
Измениться в прогрессе то, что рядом будет второе необитаемом (обитаемое) место с потенциалом на освоение. Куда долететь реально, даже если собрать корабль из говна и палок
Аноним 15/10/21 Птн 12:03:32 #146 №686037 
>>686019
Ну как минимум после изобретения радио - точно бы обнаружили, а вообще раньше - после изобретения телескопа
Аноним 15/10/21 Птн 12:11:08 #147 №686039 
>>686025
>сколько дров понадобится для сжигания и перелёт на воздушном шаре
Вопрос возник: могут ли существовать каменные планеты с общей атмосферой?
Аноним 15/10/21 Птн 12:16:21 #148 №686040 
>>686027
Потому что масса будет слишком большой. Но вообще странную хуйню ты скопипастил, в 10 раз тяжелее Юпитера это уже около-карлик.
Аноним 15/10/21 Птн 12:25:29 #149 №686043 
>>686039
Могут, в течении пары минут, потом они станут одной планетой с очень горячей атмосферой.
Аноним 15/10/21 Птн 12:29:39 #150 №686045 
>>686043
Короч на стабильных орбитах перетекание газа невозможно из-за большого расстояния и малого притяжения?
Аноним 15/10/21 Птн 12:34:42 #151 №686047 
1280px-PIA01478InteriorofCallisto.jpg
Если на Земле с продвижением вниз температура растёт на сколько-то там градусов с каждыми сколько-то там метрами, то почему мы не предполагаем, что где-нибудь на Каллисто не так же?
>>686040
>Потому что масса будет слишком большой
Эм... ну и почему при том, что масса будет слишком большой, не может быть несколько больших землеподобных планет-спутников? Да, вон у легковесного Сатурна как раз так и вышло: есть один большой спутник - Титан, а у Юпитера как раз 4 крупняка
Аноним 15/10/21 Птн 12:36:18 #152 №686048 
>>686045
С научной точки зрения атмосфера может тянуться в космос на тысячи километров, но с практической уже на ста километрах свежий бодрящий вакуум.
Аноним 15/10/21 Птн 12:39:18 #153 №686050 
>>686047
>то почему мы не предполагаем
А мы действительно это не предполагаем? может где-то есть ученый который всю жизнь строит графики температуры на Каллисто, просто знают о нем три с половиной астронома.
Аноним 15/10/21 Птн 12:42:17 #154 №686052 
>>686047
>на Каллисто не так же
Потому что она не греется из-за приливных сил.
>ну и почему при том, что масса будет слишком большой, не может быть несколько больших землеподобных планет-спутников?
Потому что в твоей копипасте сказано:
>Есть ещё один немаловажный закон природы: суммарная масса спутников гиганта в 4000-10000 раз меньше массы самого гиганта
Аноним 15/10/21 Птн 12:54:36 #155 №686054 
Spectralofdoublestars.gif
980px-RocheLobesDetailed.svg.png
>>686039
>Вопрос возник: могут ли существовать каменные планеты с общей атмосферой?
Я тебе скажу по секрету: не так давно открыли, что Луна вращается в поле атмосферы Земли. Точнее, не совсем самой Земли, а, скорее, атмосферы общего центра масс, но вот только такая общая атмосфера системы Земля-Луна оооочень разряжена, и обнаружить её можно только издали и только спецэлектроникой (как, собственно, её и обнаружили)
Другое дело, тебя интересует куда более существенная атмосфера, так? Предполагаю, что возможна, при условии, что:
1) Две планеты достаточно далеко от своей звезды, что звёздным ветром прямо не сдувало
2) На обе планеты изначально налито больше атмосферы, как воды в ванной, из-за чего она тупо расположена выше своими плотным слоями
3) Обе планеты должны вращаться по максимально возможно круговой орбите друг относительно друга с минимумом расстояния апоцентра и перецентра взаимных орбит (гифрилейтед)
4) Общая сверхразряженная атмосфера, подобная общей системы Луна-Земля находится в области полости Роша, а плотная атмосфера у обеих планет, из-за взаимного тяготения, выглядит как песочные часы, с соприкосновением в L1 (пикрилейтед)
Как-то так.
Так же предполагаю, что должен быть какой-то закрепляющий фактор в этой L1 (как мне кажется), что-то типа схождения электромагнитных полей, помимо гравитации, не дающих плотной атмосфере растекаться, но это сугубо моё предположение, ибо о электромагнитных механизмах атмосферного сдерживания наверняка, лично мне не известно
Аноним 15/10/21 Птн 12:58:38 #156 №686055 
>>685225 (OP)
почему всякие дурачки по обе стороны окияна уверены, что воду/лед с Луны можно будет гнать на ракетное топливо и не только в промышленных масштабах для всего человечества? Там этой воды меньше, чем в каком-нибудь технологическом озере, пару раз старшип заправить и месяц экипажу на кислородном пайке посидеть. Не пора ли бить тревогу? Вода Луны должна оставаться на Луне, все эти разговоры на самом деле это потенциальное изнасилование маленьких девственных недр нашего единственного спутника.
Аноним 15/10/21 Птн 13:27:21 #157 №686060 
>>685225 (OP)
Анонии, тупой вопрос от меня. Что надо учить для поступление в ВУЗ, факультет по астрономии? Мне 30, хочу получить второе образование, и почему то тянет именно в сторону астрономии
Аноним 15/10/21 Птн 13:57:40 #158 №686068 
Каждый объект во вселенной вне зависимости от размера создает гравитационные возмущения или нет?
Аноним 15/10/21 Птн 14:02:09 #159 №686069 
>>686055
Ты ещё спроси, почему всем похуй на потенциальный синдром Кесслера от низкоорбитальных группировок типа старлинка, состоящих из стопицот перекрещивающихся по прямым углом орбит.
Потому что дурачки, ёбаный-ебать.
Аноним 15/10/21 Птн 14:04:56 #160 №686070 
>>686060
>Что надо учить для поступление в ВУЗ, факультет по астрономии?
Физика, математика.
>почему то
Потому что ты долбоёб, вестимо. Люди обычно второе высшее получают, чтобы расширить или углубить свои существующие компетенции, бизнес-менеджемнт например.
Аноним 15/10/21 Птн 14:06:47 #161 №686071 
>>686068
Каждая частица имеющая массу. Но чтоб реально возмущать, массы должно быть много, ибо гравитация очень слабая сила.
Аноним 15/10/21 Птн 14:58:45 #162 №686090 
>>686070
>>Физика, математика.
Спасибо, анон! Но все таки, надо знать школьную программу?

>>Потому что ты долбоёб
Вполне может быть, но "бизнес-менеджемнт" и от всего такого блевать тянет
Аноним 15/10/21 Птн 15:04:24 #163 №686094 
>>686068
Каждый объект, обладающий ненулевой массой, искривляет пространство, а следовательно, обладает гравитацией.
Аноним 15/10/21 Птн 15:09:23 #164 №686097 
>>686090
>надо знать школьную программу
Для вступительных её должно хватить, по идее. Можешь ещё занюхнуть геологию, химию, радиотехнику, электротехнику, вот это всё.
Аноним 15/10/21 Птн 23:37:56 #165 №686179 
Если человек на стабильной орбите земли, бросит булыжник, этой энергии хватит чтобы летать вокруг Земли? или он упадет?
Аноним 16/10/21 Суб 00:46:00 #166 №686183 
>>686179
>Если человек на стабильной орбите земли
То он и так летает вокруг земли, и камень вместе с ним. Ему придётся приложить дохуя энергии, чтобы он упал.
Аноним 16/10/21 Суб 01:21:10 #167 №686184 
>>686070
>Люди обычно второе высшее получают, чтобы расширить или углубить свои существующие компетенции, бизнес-менеджемнт например.
Какая разница, что люди делают обычно? У меня первое образование переводчик, скоро отправляюсь делать второе для биотеха. Интересы поменялись, я другой человек, зачем углублять существующие компетенции?

29 лет
Аноним 16/10/21 Суб 01:32:28 #168 №686186 
>>686060
А результат какой хочешь получить на выходе?, для чего тебе вышка по астрономии?, работать в рашке такое себе занятие в этом направлении, все деньги и крутые космические проекты в европе и америке
Аноним 16/10/21 Суб 02:21:30 #169 №686189 
В 2009 году Педро Ласерда из университета Королевы в Белфасте сообщил об обнаружении на поверхности Хаумеи цветовой аномалии — крупного пятна, обладающего красным оттенком в видимом диапазоне. Природа и происхождение данного объекта точно не установлены. Предположительно, это может быть область скопления неких минералов или органических соединений
Аноним 16/10/21 Суб 07:01:39 #170 №686206 
>>685225 (OP)
Если оказаться на дневной стороне плутона, буквально стоишь в скафандре и смотришь вдаль, насколько там светло бы было?
Человек бы вообще что-нибудь рассмотреть смог?
Аноним 16/10/21 Суб 10:29:56 #171 №686219 
>>686206
Как в комнате со слабой лампочкой. Освещённость падает квадратично с ростом расстояния, насколько помню, расстояние от солнца до Плутона в среднем 40ае, а освещённость на орбите Земли, пишут в интернете, 135клк, итого получается где-то 85лк.
Аноним 16/10/21 Суб 10:38:44 #172 №686222 
Когда уже найдут на Марсе руины протеан, а ретранслятор очистят от налипшего Харона?
Аноним 16/10/21 Суб 11:26:34 #173 №686230 
>>686222
В Солнечной системе могут найти разве что простейших в океанах подледных и то не факт

На Марсе все аналогично - была вода, реки, песочек - а развитой жизни нет
Аноним 16/10/21 Суб 11:29:46 #174 №686232 
И имхо, Марс сначала нужно ебнуть чем то большим и только потом отправлять туда марсоходы.
Это нужно для того чтобы ударная волна сдвинула массу пыли и говна и обнажила более ранние слои
Аноним 16/10/21 Суб 14:51:52 #175 №686271 
>>686186
>>А результат какой хочешь получить на выходе?
На выходе, хочу получить вышку по астрономии. Конечно, в плане карьеры и денег - это плохой выбор, в этом ты прав. Но черт возьми, тянет туда и все
Аноним 16/10/21 Суб 15:46:49 #176 №686284 
>>686271
>Но черт возьми, тянет туда и все
Все правильно делаешь. Только хорошо учи английский и не замыкайся в отечественном астросообществе. Пригодится вне зависимости от того, будешь ли работать заграницей.
Аноним 16/10/21 Суб 16:04:28 #177 №686293 
что будет если прилететь на плутон, просверлить небольшую дырку, сунуть в неё флешлайт и выебать?
Аноним 16/10/21 Суб 16:10:03 #178 №686295 
image.png
image.png
image.png
Чем фото этой экзопланеты хуже фото Плутона? Почему мы не можем по таким фото заранее предсказать как будет приблизительно выглядеть поверхность экзопланеты, как это вполне успешно предсказали у Плутона?
Аноним 16/10/21 Суб 16:28:55 #179 №686303 
>>686293
Хуй отвалится, как только ширинку расстегнёшь.
Аноним 16/10/21 Суб 16:32:04 #180 №686304 
20180425pluto-comparison-bordered.jpg
nh-apluto-wide-9-17-15-final.0.png
BluehazesoverbacklitPluto.jpg
>>686295
А что там предсказали у Плутона по карте фоток поверхности? Да практически ничего. Почти все модели - планетологических, атмосферных и т.п. - не пользовались картами поверхности, потому что наилучшая реконструкция даже (не фото) была как пикрил, и можно было с таким же успехом гадать на кофейной гуще.

А New Horizons засветил и хайрез геологию и атмосферу, в результате сразу же можно стало построить атмосферную модель, вывести фотохимию, разобраться в геологическом строении тела, всё это с высокой точностью. После пролёта сразу пошли нахуй все три существовавших атмосферных модели, карты ветров (были и такие), и т.п. Плутон оказался совсем не таким, как его представляли по 3.5 мыльным пикселям из телескопя.

А у тебя на фотке вообще пиздец, ничего нет. И на первой фотке у тебя протопланетный диск (огромный объект) с по сути точечным газовым гигантом, на второй тоже огромные планеты в несколько пикселей. Как ты собираешься гадать что-то осмысленное по этим крупицами инфы? Вообще, в астрофизике и планетологии есть очень большая доля гадания на кофейной гуще, т.к. нет возможности поизучать вблизи.
Аноним 16/10/21 Суб 16:36:49 #181 №686309 
XRYfym8.jpg
>>686304
Ну и ещё простейший пример - комета чурюмого геросемёна. Реконструкция по телескопу vs наблюдение вблизи Розеттой. По кривым блеска реконструировали форму и предполагали что это такой пельмень-шестерёнка, а реально это оказались две слипшихся говняхи с намотанным между ними перешейком.
Аноним 17/10/21 Вск 02:03:04 #182 №686379 
Нахуя все хотят бомбить опасные для земли астероиды атомными зарядами? Каким образом астероид изменит свою орбиту или вообще развалится, если атомное оружие в космосе бесполезно? Ведь никакого взрыва не будет, поскольку плотность среды мала, ударной волны не будет. Энергия, выделяемая при взрыве, тупо уйдет в излучение (тепловое/свет/радиация/подобное). Я чего-то не знаю или все вокруг реально долбоебы, раз рассматривают это на серьезных щщах?
Аноним 17/10/21 Вск 02:31:57 #183 №686380 
>>686379
Да нет таких крупных опасных камней, земля под крышей уважаемого Юпитера Газонюховича
Аноним 17/10/21 Вск 03:15:34 #184 №686381 
Screenshot (164).png
>>686379
Ну если в космосе рядом взрывать - разумеется нихуя не будет. Если шахту прохуярить и в шахте взорвать, особенно направленным взрывом - можно выбросить достаточное количество массы, чтобы сдвинуть. Потому что на определенных участках траектории достаточно небольшого влияния, чтобы получить большой разлёт. Основная проблема - определить центр масс и распределие вещества внутри каменюки.

>>686380
https://cneos.jpl.nasa.gov/ca/
Камней за сотню метров, которые пролетят в будущем внутри орбиты Луны - завались. Это лишь те, которые обнаружены и у которых достаточно точно предсказанная орбита, а есть ещё те, у которых траектория плохо предсказывается, а ещё не обнаруженные.
В целом в Землю постоянно (по геологическим меркам) херачат астероиды, достаточные чтобы устроить Бейрут какой-нибудь стране.
Аноним 17/10/21 Вск 04:18:00 #185 №686382 
Что за хуйня с яркостью?
Мы находимся в галактике. Ее видно невооружённым взглядом, это понятно. Но она же тусклая что пиздец.

Просто ьлять вдумайтесь!
Телескопы могут снимать далёкие галактики с яркими ядрами, но при этом мы находимся в своей В ТЕМНОТЕ. Это вообще как?

По такой логике она должна 24/7 ебошить светом как от ядра, так и от всех звёзд в диске. Но этого нет.
Аноним 17/10/21 Вск 04:19:40 #186 №686383 
>>686382
400 МИЛЛИАРДОВ ЗВЕЗД В ОДНОЙ ГАЛАКТИКЕ + ЯРКОЕ ЯДРО

Почему темно?
Аноним 17/10/21 Вск 05:01:39 #187 №686387 
>>686382
Очевидно телескописты следят чтобы свет от более ярких объектов не засвечивал более тусклые, делают множество снимков чтобы выловить хоть какие-то фотоны от далеких объектов и т.п.
Аноним 17/10/21 Вск 05:08:29 #188 №686390 
>>686383
Этого не достаточно поскольку эти 400 миллиардов звезд рассеяны в огромном объеме.
Аноним 17/10/21 Вск 05:09:32 #189 №686391 
>>686382
>>686383
Потому что галактика размазана по всему небу. Центр вообще пылью закрыт, и в оптическом диапазоне не виден.

А если ты ещё чуть-чуть обобщишь свой вопрос, то изобретёшь фотометрический парадокс.
Аноним 17/10/21 Вск 08:34:26 #190 №686403 
Нахой нужен этот ваш космос, ваще?
Моего днищеобразования из 10 классов и технической вышки вполне хватает, чтоб понять, что текущих достижений науки не хватит на то, чтобы улететь куда-либо за пределы солнечной системы. Ресурсы таскать с соседних планет тоже не получится. Всё равно, что пиздовать за хлебом в соседний город.
Никакого профита в космосе нет, кроме набора раейтинга у пропопулистского быдлоизберателя.
Нахуй нужен космос?
Спутники - моё почтение.
Телескопы - заебись.
Космонавтика - говно без задач за деньги налогоплательщика.
Аноним 17/10/21 Вск 08:48:38 #191 №686405 
>>686383
Расстояния пиздец какие большие. Даже ближайшие звёзды типа Проксимы Центавра или звезды Барнарда, до которых всего 5 световых лет, неразличимы невооружённым глазом, наше Солнце уже с 50 световых лет будет невидимо, и только ярчайшие сверхгиганты остаются видимыми с тысячи световых лет. А ты там чё-то со 100 тысяч хочешь увидеть.
Аноним 17/10/21 Вск 10:30:53 #192 №686418 
>>686403
потому что это просто охуенно.жпг
Аноним 17/10/21 Вск 12:41:03 #193 №686446 
Из спутниковой тарелки и старого тв ресивера можно что то сделать?
Аноним 17/10/21 Вск 13:44:44 #194 №686453 
>>686446
*Я имел в виду, можно ли сделать что то васянское на этой основе и слушать ближний космос?
Аноним 17/10/21 Вск 17:01:37 #195 №686489 
>>686453
Только если сигналы от спутников или отраженные помехи от микроволновок. Заебешься сигнал фильтровать от техногенных помех.
Впрочем если не лень и у тебя есть дохуя свободной площади, то можешь собрать фазированную решетку из диполей или хотя мы жиденький интерферометр. С ними можешь послушать Юпитер или Солнечную корону. На определенных диапазонах при удаче можешь поймать сигналы активных галактик или остатков сверхновых.
Аноним 17/10/21 Вск 17:12:56 #196 №686491 
>>686446
В инете есть как принимать картинку с погодного спутника на конструкцию из говна и палок.
Аноним 17/10/21 Вск 17:34:32 #197 №686494 
>>686491
Ну если ты про ту статью говоришь, то там ровно 3 шага
1. Я воткнул проволоку в рацию
2. Подсоединил к аудио входу с пк
3. Принимаю изображения

Никаких подробностей, что и как, ничего
Аноним 17/10/21 Вск 17:37:50 #198 №686495 
>>686379
Кто все-то блять? Голивудские режиссёры?
Аноним 17/10/21 Вск 17:44:40 #199 №686497 
>>686494
https://duckduckgo.com/?q=how+to+receive+noaa+satellite+imagery
Аноним 17/10/21 Вск 23:14:30 #200 №686546 
Почему бы не установить на МКС ионные движки? Хоть они и слабые, пусть работают постоянно, энергия есть от панелей. Разве это не позволит сэкономить на массе доставляемого топлива?
Аноним 17/10/21 Вск 23:20:30 #201 №686548 
>>686546
Чел она вся в дырках, разваливается и падает на Землю, какие ионные движки
Там пробоины СКОТЧЕМ заклеивают
Аноним 17/10/21 Вск 23:22:13 #202 №686549 
>>686546
Так вроде давно поставили и много раз тестили. Но для мкс есть проблема со сбрасыванием лишнего собственного момента-импульса, который нужно изменять разом и быстро. Тут уже ионником не обойдешься.
Аноним 18/10/21 Пнд 01:12:10 #203 №686566 
15715476590512.jpg
>>686548
>Там пробоины СКОТЧЕМ заклеивают
Жеваным хлебом пусть попробуют.

>>686549
Точно, вспомнил, читал где-то про сброс импульса.
Сейчас у них там еще та свалка оборудования.
Аноним 18/10/21 Пнд 01:43:33 #204 №686568 
>>686546
Хотели VASIMR для поднятия орбиты присоединить к МКС. Но не срослось.
А вообще ЭРД жрут электричества дохуя. Там большие панели, но лишнего электричества не то чтобы завались.
Аноним 18/10/21 Пнд 02:38:21 #205 №686570 
Что будет с космонавтом, если он выйдет на обслуживание станции, отцепиться и полетит на Землю?
Аноним 18/10/21 Пнд 03:11:23 #206 №686573 
>>686497
>duckduckgo
Ебало

>>686570
> Что будет с космонавтом, если он выйдет на обслуживание станции, отцепиться и полетит на Землю?
В масс эффект 2 не играл что-ли?
Аноним 18/10/21 Пнд 07:58:05 #207 №686584 
>>686573
Гуглозависимый игроман не смог не кукарекнуть.
Аноним 18/10/21 Пнд 14:11:14 #208 №686659 
>>686573
Если ты так дрочишь на Масс эффект то должен был знать, что Шепард упал не на планету земной группы
Аноним 18/10/21 Пнд 14:11:57 #209 №686660 
>>686570
За счёт чего полетит?
Аноним 18/10/21 Пнд 14:14:58 #210 №686661 
image.png
Ну и я собственно дополню вопрос, если на мой первый все равно никто не ответил

Допустим космонавт все таки отцепился, и начал свое свободное падение на Землю, допустим что кислород у него на максимуме.
Начал он свое падение потому что например произошел ТОЛЧЕК.
Точка предполагаемого падения - тихий океан

Насколько для него будет критичным вход в атмосферу? А если на скафандре будут подкрылки?
Аноним 18/10/21 Пнд 14:27:42 #211 №686668 
>>686661
Я тебе вопрос задал. За счёт чего он падает на Землю, что дало ему импульс и в какую именно сторону? Если он просто руками оттолкнулся от стенки станции в сторону Земли, то первые минут двадцать он будет медленно отдаляться от станции в сторону Земли и чуть-чуть вперёд по направлению полёта станции, следующие двадцать минут он будет двигаться от Земли уже вверх, но всё ещё вперёд по направлению полёта. Затем он начнёт двигаться обратно против полёта станции и всё ещё вверх, пройдёт над станцией и окажется позади неё приблизительно через 90 минут после того, как оттолкнулся.
Аноним 18/10/21 Пнд 14:33:10 #212 №686669 
>>686661
Если как ты показал, то он сгорит.
Аноним 18/10/21 Пнд 14:47:24 #213 №686670 
>>686669
Помянем
Аноним 18/10/21 Пнд 15:57:37 #214 №686681 
Анон, почему до сих пор не нашли девятую планету? Батыгин - красавчик? Гипотеза хорошая?
Аноним 18/10/21 Пнд 16:21:41 #215 №686683 
>>686681
Никакой девятой планеты нет
Аноним 18/10/21 Пнд 16:31:43 #216 №686684 
>>686681
Ищут. Недавно ожили >>676765 → , но толком ничего не сказали - критерии отфильтрованы, наблюдения идут.
Гипотеза всегда была слабее остальных трёх, просто распиарена до невозможности. Там есть ещё "седноиды сами так построились" (буквально, там уберхитрая орбитальная механика от японцев, я сам до конца не понял), влияние резонанса больших планет, и близкий пролёт крупной звезды в прошлом. Но они все тоже слабые. Кроме того, точно даже неизвестно, проблема ли это вообще, может седноиды везде, а не только с одной стороны, просто мы ещё не нашли остальные. Так что там всё может оказаться миражом на мираже. Нужны ещё многие годы наблюдений.
Аноним 18/10/21 Пнд 16:57:55 #217 №686685 
82853870438035353.pdf-55.jpg
>>686661
Во-первых, чтобы сойти с орбиты следующим витком, толчок должен быть в направлении обратном орбитальному движению, отшырнув его метров где-то на 100 в секунду. Но допустим это произошло и его не расплющило. Тогда он просто сгорит в атмосфере нахуй.

Просто задумайся, что на вывод на орбиту ты сжёг топливо в огромной ракете, сколько там было энергии - вот всю её тебе придётся рассеять по сути собственной жопой об воздух.

>А если на скафандре будут подкрылки?
Под что???

Вещь которую ты ищешь, но не знаешь что оно тебе надо: low-temperature reentry, низкотемпературный вход в атмосферу. В типичном случае нужен надувной конический парашют, его необходимо спускать по очень пологой траектории (едва касаясь плотных слоёв, но не так чтобы отпружинить и входить скачками), под неким углом атаки (чтобы была подъёмная сила). Да, он должен быть покрыт чем-то вроде кевлара, или абляционки, потому что температуру входа ниже 350 градусов вообще нереально получить. Вот такой воланчик - самое реалистическое представление "орбитального парашюта".

В 60х и 70х рисовали концепты орбитальных парашютов, но они все наркоманские были. В 90х НТУ реально вошёл в атмосферу и выжил при этом. NASA в 2010х прорабатывала и до сих пор изучает такие воланчики для торможения в атмосфере Марса. Основная плохо решённая проблема у них это управляемость движения и точность посадки, а также отвод тепла излучением.

Есть ещё вариант с тросом, тоже наркоманский. За объектом разматывается тонкий стальной трос 20км длиной. Он автоматически стабилизируется градиентом гравитации и при входе создаёт нехилое сопротивление воздуха, тормозя объект. Вопросов в такой схеме больше чем ответов.
Аноним 18/10/21 Пнд 17:33:29 #218 №686692 
>>686685
Как это под что? Трение обо что по твоему создается? Об атмосферу. Ведь тело в нее ВХОДИТ.
Так может и подкрылки будут не бесполезны? а позволят немного скользить, снижая скорость входа
Аноним 18/10/21 Пнд 18:00:32 #219 №686696 
>>686681
В свете последних данных вероятность существования девятой планеты не такая уж и большая. Не так давно выходила статья с критикой гипотезы о девятой планете. Там вывод такой, что очень может быть, что никакой девятой планеты нет. https://phys.org/news/2021-02-evidence-planet-diminishing-clustering.html Вот тут тоже говорят о том, что девятой планеты может не быть https://earthsky.org/space/mystery-orbits-not-caused-by-planet-9-researchers-say/ Короче, планета может быть и есть, но вероятность ее присутствия оценивается сегодня как гораздо меньшая, чем когда эта гипотеза только появилась и просочилась в массы.
Аноним 18/10/21 Пнд 18:45:01 #220 №686700 
mhobqfpw8x101.jpg
Попаданцев тредю. Ты обладаешь условным бессмертием и оказываешься в космосе, на расстоянии 330 световых лет от Земли - твои действия?
Аноним 18/10/21 Пнд 18:52:10 #221 №686701 
>>686700
Условно пукнув, просыпаюсь на лекции. Слишком размытые начальные условия.
Аноним 18/10/21 Пнд 19:00:48 #222 №686702 
>>686692
Это был тонкий намёк что подкрылки - это у автомобиля под крыльями. У летательного аппарата или капсулы никаких "подкрылков" нет, блеать. У него есть крылья.

Воланчик как раз и скользит на ненулевом угле атаки, создавая подъёмную силу. Это и есть твой "подкрылок", лол. И подъёмная сила нужна вовсе не для снижения скорости, а для того чтобы сразу не зарыться в атмосферу и стравливать скорость медленно, размазав её потерю по более длинной траектории, тогда нагрев будет не таким быстрым и ты не наберёшь такую высокую пиковую температуру.

Тащемта все современные аппараты, включая капсулы, никогда не пользуются баллистическим спуском (лишь в аварийном режиме это предусмотрено). Капсулы спускаются тоже под углом, со скольжением. Проблема в том, что на глубоком гиперзвуке практически любая форма имеет очень хуёвую подъёмную силу - что крыло, что кирпич летают одинаково плохо на таких скоростях. Воланчик хотя бы имеет большую площадь относительно массы, поэтому имеет высокий баллистический коэффициент, поэтому такая форма выгодней для медленного спуска.

Обратная сторона медленного торможения - общий теплопоток, проходящий через аппарат, намного выше чем с баллистическим спуском. Поэтому тепло надо куда-то излучать, нужен радиатор.
Аноним 18/10/21 Пнд 19:05:57 #223 №686703 
>>686701
А вот свобода действий и обстоятельств даются тебе
Аноним 18/10/21 Пнд 19:05:59 #224 №686704 
>>686700
Пукаю и придаю себе движение в сторону земли
Пока долечу решат проблемы голода, перенаселения, всем будет довольствие как в эмиратах

или фоллаут 2
Аноним 18/10/21 Пнд 19:06:52 #225 №686705 
>>686704
А как направление к Земле узнаешь?
Аноним 18/10/21 Пнд 19:07:51 #226 №686706 
>>686705
Ну я же до этого как то оказался там?
Соориентируюсь
Аноним 18/10/21 Пнд 19:11:57 #227 №686708 
>>686706
Нет, ты внезапно там респаунишься
Аноним 18/10/21 Пнд 20:34:13 #228 №686713 
>>686700
Очевидно пуками добираюсь до сюда:
HD 330 — звезда, которая находится в созвездии Кассиопеи на расстоянии около 332 световых лет от нас, звезда F-класса 8,151 величины, она имеет массу, равную 1,12 массы Солнца. Возраст звезды оценивается приблизительно в 5,1 миллиардов лет. Планет в данной системе пока обнаружено не было.
Аноним 18/10/21 Пнд 20:45:33 #229 №686715 
>>686703
Ну, то есть ты даже не удосужился придумать что-то, что могло бы кого-то заинтересовать, а я должен начать тебя тут развлекать при всех? Не, не интересно.
Аноним 18/10/21 Пнд 20:46:52 #230 №686716 
>>686685
>Есть ещё вариант с тросом, тоже наркоманский. За объектом разматывается тонкий стальной трос 20км длиной. Он автоматически стабилизируется градиентом гравитации и при входе создаёт нехилое сопротивление воздуха, тормозя объект.
Это как? Как это работает? Я не понимаю. Объясни на пальцах.
Аноним 18/10/21 Пнд 20:52:02 #231 №686717 
>>686716
https://www.sciencedirect.com/science/article/abs/pii/009457659500108C
Трос многокилометровый, и у него достаточная площадь поверхности, чтобы создавать дохуя трения, и тут же на месте рассеивать созданное тепло.
Аноним 18/10/21 Пнд 21:21:10 #232 №686722 
2a54b4db8a3223e0d8742852b5bed1c5[1].jpg
>>686717
А как эти 20км проволоки я, гипотетически падающий с орбиты, должен буду с собой брать и как раскручивать? Что-т мне подсказывает, что скрюченный провод в невесомости не будет распрямляться и я, при попытке его раскрутить, скорее превращусь в пикрил. Что значит, что при падении в атмосферу металлические нити, окутавшие моё тело, вместо спасательных станут для меня губительными, так как весь жар буду передавать прямо на моё тело.
И рассуждая так, в чём я не прав?
К тому же, каково должно быть 20км металлического проволоки? Она сама не превратится в абляционную смесь из-за аэроторможения и температур и не расплавится в полёте? А прикинь, если у такой нити произойдёт разрыв не где нибудь на 19км от тела приземлятора, а в метрах 100 от него? И тогда все 19,9км металлической проволоки полетят следом за падающим мужиком (с кем не бывает, хули), а он, стало быть, просто будет падать. Без охлаждения и тросового парашюта.
Аноним 18/10/21 Пнд 21:33:58 #233 №686727 
YES2Baikonurweek1.2pillars.jpg
>>686722
>А как эти 20км проволоки я, гипотетически падающий с орбиты, должен буду с собой брать и как раскручивать? Что-т мне подсказывает, что скрюченный провод в невесомости не будет распрямляться и я, при попытке его раскрутить, скорее превращусь в пикрил.
Например многожильный трос достаточной гибкости или с памятью. Отстрел балансира с привязанным концом, пружиной. А дальше, при выпуске на достаточную длину - градиент гравитации его сам уложит в нужном направлении и перевесит стремление троса скрючиться. Общие принципы тросовых систем в 70х ещё были отработаны.

Такой эксперимент с тросовым реентри-парашютом проводили в YES2/Фотино в 2007 году, попуткой с Фотоном-М3. Там хотели размотать трос на 30км, но размотали только на 8.5 и аппарат сломался. это был кубсат, литералли сделанный студентами
Правда там был арамидовый (кевларовый) трос толщиной 0.5мм, вот такая бобина. Но стальной был бы с такой же гибкостью.
Аноним 18/10/21 Пнд 21:37:02 #234 №686728 
>>686722
>Она сама не превратится в абляционную смесь из-за аэроторможения и температур и не расплавится в полёте?
Для этого надо просто посчитать весь входящий и выходящий теплообмен. В статье выше это сделано с 1мм стальным тросом. 20-30км это очень много, достаточно чтобы тот не расплавился и пиковая температура оказалась низкой. Здесь >>686727 вообще с полимерным посчитали и всё сошлось, правда эксперимент провести не удалось.
Аноним 18/10/21 Пнд 21:54:44 #235 №686729 
>>685225 (OP)
Почему никто не спасает всяких застрявших альпинистов на эверестах с помощью вертолетов? Трупы бы там прибрали и похоронили например.
Аноним 18/10/21 Пнд 22:12:08 #236 №686737 
>>686729
1) Лютые и непредсказуемые ветра, которые могу налететь, прекратиться и начать дуть в другом направлении
2) Пурга, снежная пыли и даже туманы ооочень мешают обзору, и я про день говорю. В сумерки и в ночь там делать нечего.
2.б) Эти самые снега силою ветра скрывают по собой трупы, вещи и целые палаточные лагеря. тупа ненаход
3) Трупы лежат не на удобных открытых воздуху и вертолётах склонах, а в закрытых щелях, расщелинах или непролазных даже для человека трещинах, куда вертолёту тем более не добраться
Аноним 18/10/21 Пнд 22:59:31 #237 №686753 
Что будет если в планете где тектоническая активность остановилась - проделать сквозное отверстие? И прыгнуть туда.

Человек пролетит насквозь или застрянет в центре?
Аноним 18/10/21 Пнд 23:17:28 #238 №686757 
>>686753
Ну если для тебя даже тектоническая активность важна, то отверстие схлопнется под действием силы тяжести. А если тебя тупо физика интересует, то попробуй найти силу, которая бы его там остановила.
Аноним 18/10/21 Пнд 23:18:17 #239 №686758 
>>686757
*тупо физика полёта
Аноним 19/10/21 Втр 01:14:42 #240 №686767 
>>686700
> Попаданцев тредю. Ты обладаешь условным бессмертием и оказываешься в космосе, на расстоянии 330 световых лет от Земли - твои действия?
Рандомно дрыгаю ножками к ближайшему газовому гиганту и падаю на него. Интересно, убьет ли меня давлением.
Аноним 19/10/21 Втр 08:34:31 #241 №686783 
>>686489
Можно подробнее?
Аноним 19/10/21 Втр 12:44:49 #242 №686807 
>>686713
Прилетаешь, а там кучас астероидных дисков.
Попа болеть не будет?
Аноним 19/10/21 Втр 18:30:06 #243 №686831 
>>686767
Пердеть надо.
Аноним 19/10/21 Втр 18:40:55 #244 №686832 
>>686783
Сначала внятно опиши, что ты хочешь и что у тебя для этого уже есть, какой бюджет на остальное. Если ты даже этого не можешь сделать, о каких самодельных фазированных решетках вообще может идти речь?
Аноним 19/10/21 Втр 18:47:35 #245 №686833 
>>686737
что то мне подсказывает что 8 км для вертолета это нихрена не штатная работа вообще, скорее превозмогание и рекорды из разряда "я так могу, сбросив все лишнее". Тут нужен специальный, под это дело разработанный вертолет.
Аноним 19/10/21 Втр 18:49:16 #246 №686834 
>>686833
Кмк, вертушка вообще не поднимется на такую высоту из-за разряженного воздуха.
Аноним 19/10/21 Втр 20:38:44 #247 №686856 
>>686729
>Трупы бы там прибрали и похоронили например.
Прикинь, это дорого и рисковано. Давай деньги, приберем.

>>686737
>Трупы лежат не на удобных открытых воздуху и вертолётах склонах, а в закрытых щелях, расщелинах или непролазных даже для человека трещинах
Тем более что замерзающие люди имеют обыкновение прятатся от ветра в какие-нибудь дыры и расщелины, а не выходить на открытое пространство.
Аноним 19/10/21 Втр 22:20:50 #248 №686882 
Почему нет такой услуги, как "похороны на Луне"?

Пиздатая идея же.
Труп мумифицируют и укладывают в гроб-лендер весом в тонну.
Он приземляется на Луну (можно выбрать понравившийся район) и остаётся там на ближайшие несколько десятков тысяч лет, пока лунной цивилизации будущего не перестанет хватать свободного места или не ебанет метеоритом.
На гроболендере устанавливается радиомаяк, периодически вещающий данные о покойнике, чтобы люди на земле могли благоговеть перед его величием.
Очень красиво же, будущий клиент может быть уверен, что его труп будет бережно мумифицирован вакуумом и сохранен на тысячи лет в месте с прекрасным видом.

За один пуск можно отправлять сразу несколько гробов.
Также можно замутить услугу для небогатых людей: сжигание в крематории и отправка на Луну только колбы с прахом в составе коллективного склепа-лендера. При полезной нагрузке лендера в 100 кг, можно отправить тысячу колб по 100 грамм.
Хорошая идея для стартапа. Такое погребение будет стоить всего 10-50к долларов, в развитых странах многие смогут себе позволить.
Пока не набралось достаточно колб для отправки, они спокойно хранятся на складе компании.

Также можно развивать и другие направления: Марс (самое очевидное), Венера, сжигание в атмосфере Юпитера (для сторонников полной аннигиляции из этого мира), орбитальное кладбище, запуск в направлении других звёзд и т.п.

Аноним 19/10/21 Втр 22:23:01 #249 №686883 
c80d19255a1591dda26e2b752c4cba2772cb6906.png
Что будет с часами в открытом космосе?
Аноним 19/10/21 Втр 22:44:47 #250 №686888 
>>686882
Чел труп весит столько же сколько и живой (почти)
Какие нахер богатые и не богатые. Единицы согласились потратится на обычный полет, а ты про отправку мертвых тел непонятно куда.

Аноним 19/10/21 Втр 22:50:49 #251 №686889 
>>686882
На такую хуиту никто не рвскошелится.
Аноним 19/10/21 Втр 23:36:56 #252 №686892 
Как примерно выглядит нейтронная звезда? Есть видео?
Аноним 20/10/21 Срд 00:05:38 #253 №686894 
>>686883
Смотря какая орбита. Если попадает в тень Земли, батарейка замерзнет и часы остановятся, пока не выйдут из тени. Так несколько раз пока батарейка совсем не сдохнет от перепадов температуры. С самими часами вряд ли что-то будет, там ломатся нечему, разве что со временем пластик деградирует от УФ.
Аноним 20/10/21 Срд 00:06:45 #254 №686895 
>>686892
Никто не знает. Ее невозможно рассмотреть ни с телескопов (мелкий размер), ни вблизи (мгновенная прожарка).

И даже если бы ты оказался рядом с ней то она во первых фонит чем только возможно, во вторых искривляет фотоны.
Аноним 20/10/21 Срд 00:07:55 #255 №686896 
>>686895
Офк я говорю о самой звезде и ее поверхности, в остальном по остаточным признакам можно определить что это звезда. Но по факту ты ее не увидишь, как увидел бы Солнце
Аноним 20/10/21 Срд 00:13:34 #256 №686897 
image.png
Самая ближайшая нейтронная звезда
Примерный радиус - 14 км
Аноним 20/10/21 Срд 00:16:14 #257 №686898 
>>686897
Это в каком диапазоне?
Аноним 20/10/21 Срд 00:42:18 #258 №686901 
>>686882
> Почему нет такой услуги, как "похороны на Луне"?
Как нет? Есть
https://www.celestis.com/experiences-pricing/luna/
https://elysiumspace.com/
Эти кооперируются с Астроботик и соответственно лишь планируют первый полёт, намечено на следующий год. А до того хоронили на орбите.
Естественно, отправляют прах, "гробы в тонну" никто пускать не будет. Тут хоть так бы бизнес развить.

Юджина Шумейкера похоронили на Луне, на борту Lunar Prospector в 99 году. Но это был не коммерческий сервис.
Аноним 20/10/21 Срд 01:15:02 #259 №686905 
>>686898
Ты даун?

В синем.
Аноним 20/10/21 Срд 01:27:37 #260 №686907 
>>686905
Ещё один лох с кривым подключением к матрице попался, высылаем зондеркоманду.
Аноним 20/10/21 Срд 03:14:11 #261 №686912 
Космос это симуляция. Весь мир это симуляция. Почему вы ещё этого не поняли?
Какая разница что там, если это "там", просто набор неопределенных данных?
Нет никакой темной материи, это просто непросчитанные за ненадобностью области.

И я хочу вам это доказать. Найдите человека, лучше в живую. Возьмите каждый по листу бумаги.
Ваша задача одновременно по сигналу воспроизвести диапазон цифр 0-9.
Порядок берется случайным образом, первое что приходит в голову. После того как закончите, просто взгляните на листы.

Не делайте этого если не готовы увидеть правду, я вас предупредил
Аноним 20/10/21 Срд 03:28:17 #262 №686913 
>>686912
>Не делайте этого если не готовы увидеть правду, я вас предупредил
В чем вообще смысл симулировать?
Аноним 20/10/21 Срд 08:18:48 #263 №686921 
>>686832
>Сначала внятно опиши, что ты хочешь
Послушать Юпитер или Солнечную корону, поймать сигналы активных галактик или остатков сверхновых.

>что у тебя для этого уже есть
Ничего

>какой бюджет на остальное
Не знаю, а какой должен быть по-хорошему?

>Если ты даже этого не можешь сделать, о каких самодельных фазированных решетках вообще может идти речь?
Я не тот >>686446 анон, мне просто стало очень интересно. Может, у тебя есть ссылка на какой-то гайд, который ты считаешь годным?
Аноним 20/10/21 Срд 09:42:39 #264 №686927 
>>686921
Я тоже не он, но насчет принять сигнал от Юпитера все не столь сложно, если по минимуму.
Допустим уже есть тарелка и LNB на Ku диапазон. Возьмем график со спектром https://ru.wikipedia.org/wiki/Магнитосфера_Юпитера В Ku диапазоне 2х10^-19 Вт/(Гц×м^2), метры тут это апертура антенны, герцы - полоса приемника, ну и получаем мощность сигнала в этой полосе. Допустим антенна 90 см с площадью 0.6 м^2, тогда в полосе 1 Гц прилетит 10log(0.6×2×10^-19)=-159 дБм. Теперь можно сравнить с шумовой температурой антенны, как пишет та же википедия https://ru.wikipedia.org/wiki/Шумовая_температура_антенны для такой антенны температура будет 30K, в полосе 1 Гц это даст -183 дБм на 1 Гц. Конечно нужно добавить коэффициент шума, но он не столь большой. Итого разница между антенной направленной в участок небо без каких-нибудь Юпитеров и на Юпитер будет около 20 дБ, что весьма заметно.
На счет помех не стоит волноваться, это Ku диапазон и параболическая антенна, сбоку ничего существенного не залетит. Но даже если так, то ничего не мешает сделать все это за городом.

Так что все просто, приделываем антенну к монтировке от телескопа, подключаем LNB к rtl-sdr, крутим антенну, и уверенно обнаруживаем Юпитер. Только в этом диапазоне Юпитер излучает более-менее равномерно, совсем не так как на более низкочастотных диапазонах, а потому и менее интересно.

Если я в чем-то ошибся, то пожалуйста не поливайте говном, а приведите правильные расчеты.
Аноним 20/10/21 Срд 10:14:01 #265 №686931 
>>685225 (OP)
Что планируют делать с космонавтикой, когда нефть кончится?
Аноним 20/10/21 Срд 10:51:48 #266 №686934 
>>686931
Водород, метан и прочие гидразины тоже кончатся?
Аноним 20/10/21 Срд 10:52:21 #267 №686935 
>>686931
Старые добрые ракеты на угольной тяге.
Аноним 20/10/21 Срд 12:17:25 #268 №686942 
>>686927
Извиняюсь, ошибся и весьма значительно. Тот график нормирован по расстоянию 1 а.е. Значит сигнал будет на примерно на 14...15 дБ слабее. Но все равно получается что для обнаружения хватит 90 см тарелки.
Аноним 20/10/21 Срд 13:23:59 #269 №686949 
>>686892
Горячий 10-километровый шарик, охуенно быстро вращающийся. Поскольку нейтронная звезда относительно горячая, она должна светить голубоватым светом.
Аноним 20/10/21 Срд 15:29:46 #270 №686970 
>>686949
>Поскольку нейтронная звезда относительно горячая, она должна светить голубоватым светом.
Она охуенно горячая относительно чего угодно. И светит не голубоватым, не синим, даже не ультрафиолетом, а ёбаной охуевшей гаммой. Лишь небольшая часть излучения оптическая, впрочем этого хватит на яркое синее свечение. Ну и не совсем шарик, т.к. искажает свет сильной гравитацией, да. Там ведь вроде должна наблюдаться анизотропия разных сторон, как у ЧД. Типа сторона вращающаяся к тебе и от тебя - будут по-разному выглядеть.

Но это всё предполагает что это НЗ без нихуя в вакууме, такую ещё поискать надо. На практике на неё будет аккрецировать какое-нибудь говно скорее всего, усложняя картину. Из-за ебовых магнитных полей аккрецирующий понос будет пидораситься в плазму и гнуться по магнитным линиям, и всё это будет напоминать светящийся герб Железногорска нахуй, ну там где медведь разрывает пасть мирному атому.
Аноним 20/10/21 Срд 15:45:01 #271 №686973 
>>686970
А ещё джеты должны быть нихуёвые, если говно на неё аккрецирует.

А при слиянии двух таких они должны высрать в джет кучу нейтронно-электронно-протонного материала. Который после потери дегенеративного давления внутри НЗ рекомбинирует обратно в тяжёлые элементы. Выглядеть это будет наверно как плазменное облачко, или типа того.
Аноним 20/10/21 Срд 22:41:48 #272 №687033 
>>686927
>>686942
Спасибо, анончик. У меня на крыше многоэтажки висят мертвые тарелки. Думаю, там можно будет что-то собрать
Аноним 20/10/21 Срд 22:42:22 #273 №687034 
>>687033
Только проверь что рядом не находится никаких военных баз, или с тарелочек сразу улетишь на бутылочки
Аноним 21/10/21 Чтв 01:33:36 #274 №687045 
>>686905
>В синем.
Понятно, то есть рядом с твоей ориентацией.
Аноним 21/10/21 Чтв 13:31:51 #275 №687094 
Поясните, если планета вращается так, что осеннее равноденствие приходится на положение орбиты планеты так, что именно в этот момент планета ближе всего к своему солнцу, то как там будет проходить смена времён года? Какие там будут зимы, весны, лета и осени?
Аноним 21/10/21 Чтв 13:54:32 #276 №687098 
>>687094
Очевидно в зависимости от того насколько ближе.
Аноним 21/10/21 Чтв 14:00:00 #277 №687100 
>>687094
>Какие там будут зимы, весны, лета и осени?
Такие же. Ведь лето это когда жарче всего, осень идёт после лета и перед зимой, зима это когда холодно, весна после зимы. Будет просто летнее равноденствие.
Аноним 21/10/21 Чтв 14:15:56 #278 №687104 
>>687098
>Очевидно в зависимости от того насколько ближе
Стандартно берём одну КакЗемлю. КакЗемля во всём, включая положение вокруг КакСолнца, орбиту, наклон, скорость вращения - всё точно такое же, только на момент перигея и апогея происходят не солнцестояния, а равноденствия
Аноним 21/10/21 Чтв 14:22:03 #279 №687105 
>>687104
У Земли орбита почти круглая и апофигей с перефигиеем не влияют на климат, он зависит только от наклона оси географии Земли, активности Сорлнца, бабочки которая крылышками бяк-бяк-бяк-бяк и т.д
Аноним 21/10/21 Чтв 16:19:05 #280 №687121 
>>687094
Смена времен года происходит из-за наклона оси вращения планеты, а не потому что она ближе/дальше от звезды конкретно Земля имеет наклон оси в 23,5 градуса и этим обусловлена смена времен года на ней.
Аноним 21/10/21 Чтв 18:59:42 #281 №687166 
>>687105
>>687121
ну вообще зависимость на земляшке имеется хоть и не решающая, в южном полушарии из-за этого лета жарче а зимы холоднее при тех же условных, когда в северном наоборот зимы приходятся на более близкое к солнцу расстояние
Аноним 21/10/21 Чтв 20:19:01 #282 №687181 
nIuaC5w4EoY.jpg
1563967835rfdnjnjkjfbf.jpg
Надо придумать как приташить астероид Психея на землю, ибо каменюка бохата железякой что даст возможность строить корабли поколений и лететь прямиком на траппист 1 и засирать миры там. Какие вообще идеи по отклонения его орбиты и как замедлить его скорость, чтобы его допустим на луну посадить или тупо тыщами зондами с рукой манипулятором-киркой долбить его с последующим доставкой ресурсов на землю?
Аноним 21/10/21 Чтв 20:31:42 #283 №687183 
Придумал себе планету-дикий запад, где есть только океанические полюса, а вся суша представляет из себя эдакую Испанию. Даже придумал, что по экватору тянется горный хребет, разделяющий полушария.
Мол, холодные воды есть только в самых полярных зонах океанов, на территория крайних северов острова с умеренной растительностью, потом ближе к суше настоящие тропические острова, потом суша, которая представляет из себя тропики только первые десятки км суши у моря, а затем растительность всё больше мельчает, превращаясь сперва в саванны, потом в жаркие солнечные степи, как в Испании, Италии или Мексике, ну а потом совсем пустыни до экваториальных гор. Ну а экваториальных горах самый ужас - высоченные горы, где небо темнеет, куда ветра не додувают дождевые облака, и где с сотню-другую км только высота плато/гор, представляющее из себя каменную пустыню, подставленную всем ветрам, выжигаемую жарким солнцем днём, и промерзающее ночью.
А потом я хотел почерпнуть быть пригорных и горных жителей такого жаркого местечка, ориентируясь на Испанскую Андорру, типа, смотрите, в каком аду живут люди. Вот только оказалось, что в реале у высокогорной Андорры, хотя она и расположена в жарой Испании, погода воистину альпийская и что там вообще нихуя не жарко даже летом.
Почему так?
Я-то думал, что в отвесных скалах не может быть ничего - мёртвая непроходимая пустыня, без жизни, без растительности. А оказывается, что при климате Андорры сама Андорра чуть ли не самая кайфовая и прохладная, чем вся остальная Испания. Я не понимаю.
Аноним 21/10/21 Чтв 21:02:00 #284 №687188 
>>687183
>что по экватору тянется горный хребет
Тогда на планете нихуя не будет примерно одинаковая температура как в Гигантской Испании. На нашей планете такая разница температур как раз из-за гор, ветра не могут циркулировать по всей планете.
Аноним 21/10/21 Чтв 22:06:25 #285 №687199 
>>687188
А если наклон планеты будет? От как на Земле до градусов, эдак, сорока?
Аноним 21/10/21 Чтв 23:54:01 #286 №687212 
>>687199
Тогда будут ветры, которые сдуют нахую эту Испанию в Солярисе, и вместо нее появится очко как на Юпитере.
Аноним 22/10/21 Птн 00:13:09 #287 №687217 
>>686901
> Как нет? Есть
Во, заебись
> "гробы в тонну" никто пускать не будет.
А это плохо, прах - программа минимум, а максимум нужно полноценное лунное кладбище
Аноним 22/10/21 Птн 02:28:09 #288 №687229 
Посоветуйте источник новостей, можно на ютубчике, можно просто в виде ньюс-фида. Суть такова: Че там у пиндосов, че там у хохолов роскосмоса, китайцев, миллиардеров. Ну в общем, кто куда полетел, кто что запланировал, кто что пукнул в свиттере.
Критерии: без предвзятости на национальной почве, на английском или русском, без разницы. Конечно, можно мониторить раздел, но это тоже так себе, слишком много всякого шитпостинга.
Аноним 22/10/21 Птн 02:57:32 #289 №687231 
>>687229
Маск всех выебал. Поккокмок сосет. Китайцы аутируют в соло.
Аноним 22/10/21 Птн 03:21:34 #290 №687233 
3b5a1415cd0ddf013bf8a1635a9dba78.jpg
>>687229
>кто куда полетел
в космос
Аноним 22/10/21 Птн 04:18:51 #291 №687236 
>>687183
Я-то думал, что в отвесных скалах не может быть ничего - мёртвая непроходимая пустыня, без жизни, без растительности
На отвесных скалах да - ни жизни ни растительности, кроме торчащих деревьев, за которые чудом цепляются упавшие в пропасть путники, и горных козлов. Но я не думаю что жители Андорры всю жизнь найтуются к страховочным крюкам и ходят за хлебушком в связке.
> А оказывается, что при климате Андорры сама Андорра чуть ли не самая кайфовая и прохладная, чем вся остальная Испания
Пиринеи не Гималаи и при испанском субтропическом климате в Андорре климат будет умеренным.
Аноним 22/10/21 Птн 04:21:03 #292 №687237 
>>687229
Канал Альфа Центавра, хоть и от расового хохла, но без предвзятости и на русском.
Аноним 22/10/21 Птн 05:31:59 #293 №687243 
подскажите немогу найти видео.
На нем по моему Кириллу Масленникову задают вопрос. Можно ли при нашей жизни увидеть планету в другой звездной системы на уровне, что можно различить скалы, материки. Он отвечает, что нет.
Хотел отписаться под видео про миссию HabEx, которая как раз это планирует
Аноним 22/10/21 Птн 06:07:37 #294 №687244 
>>687243
Это та где солнце хотят использовать вместо линзы?
Аноним 22/10/21 Птн 06:12:01 #295 №687245 
>>687243
Тык Хабекс запустят в 30-е годы, там уже не то что он доживет, пол двача ркн-ется, так что он тебя уест в этом вопросе
Аноним 22/10/21 Птн 10:56:30 #296 №687258 
безопасно ли.jpg
Получится ли безопасно упасть из космоса в воронку, заканчивающеюся параболической горкой? Воронка и горка должны погасить ускорение падения и снизить скорость, если учесть что пациент 100% попадет в воронку?
Аноним 22/10/21 Птн 10:57:23 #297 №687259 
>>687258
>пациент
*блджат, космонавт конечно же.
Аноним 22/10/21 Птн 11:06:22 #298 №687263 
>>687258
Поциент сгорит при входе в верхние слои атмосферы. Гашение скорости у земли — наименее сложная часть процедуры.
Аноним 22/10/21 Птн 11:49:25 #299 №687269 
>>687263
Если воронку сделоть в космосе, а горку очень длинной?
Аноним 22/10/21 Птн 12:15:33 #300 №687272 
>>687269
Сгорит при трении о воронку.
Аноним 22/10/21 Птн 13:32:42 #301 №687279 
image.png
>>687258
Как он в неё попадёт, блядь, если он по эллипсоиде вдоль поверхности ебашит?
Аноним 22/10/21 Птн 16:08:35 #302 №687296 
>>687258
Это подойдёт как доп услуга вертолетных прогулок для коммунистов
Аноним 22/10/21 Птн 17:19:15 #303 №687307 
image.png
>>687279
Вот теперь не промахнётся.
Аноним 22/10/21 Птн 18:12:45 #304 №687310 
>>687212
>Тогда будут ветры, которые сдуют нахую эту Испанию в Солярисе, и вместо нее появится очко как на Юпитере
Это ещё почему?
Аноним 22/10/21 Птн 18:15:48 #305 №687311 
>>687229
Космические паблики в Яндекс.Дзене
Пикабу
/news/ двача и тред космических новостей в /spc/
Аноним 22/10/21 Птн 19:12:38 #306 №687318 
>>687229
https://nplus1.ru/
Аноним 22/10/21 Птн 21:34:42 #307 №687334 
>>685225 (OP)
Сколько лететь с земли до марса с ускорении 2г на максимально м сближении и наоборот

2г туда пол пути и 2г на торможение
Аноним 22/10/21 Птн 22:16:34 #308 №687336 
>>685225 (OP)
Почему разнится картинка между сайтами https://mks.space и https://www.youtube.com/watch?v=TD-Ji-LVmn4
Вид с разных камер? Или обман гоев? Задержка картинки? Но ведь местоположение МКС на трубе берётся с сайта МКС, вроде как. Так почему такие разночтения, в стиле " на одном сайте над океаном подлетаем к к Канаде", а на втором (труба) успешно летим к её (Канады) западной границе. Или на трубе крутят снятые за сутки кадры от нынешнего положения МКС, чтобы народ развлекать? Поясните ньюфагу, будьте добры
Аноним 22/10/21 Птн 22:25:25 #309 №687337 
>>687336
На ютупе написано же, что показывают как в реальном времени, так и в записи:
>THIS WILL SHOW LIVE and PRE-RECORDED FOOTAGE - depending on signal from the station or if the ISS is on the night side of Earth.

>When the feed is live the words LIVE NOW will appear in the top left hand corner of the screen.
Аноним 22/10/21 Птн 22:37:11 #310 №687340 
>>687337
Я не очень уметь в английский, и, конечно же, не читал описание канала. Если я правильно понял, то на трубе стрим идёт, но, если случается какая-то шляпа со связью, типа попадания на тёмную сторону, где солнечные батарейки не работают, то они крутят записи предыдущих полётов. А на сайте МКС всё по-честному: нет связи или тёмная сторона - вот вам синий экран или экран, полный помех, и ждите солнышка, в общем, так? Хотелось бы понять, какому источнику информации верить-то, где залипать, чтобы быть словно здесь и сейчас в этом вашем нашем околоземном космосе и смотреть на планету онлайн без обмана
Аноним 23/10/21 Суб 06:31:41 #311 №687365 
>>686970
А если фильтруешь всю гамму и просто воссоздаешь тупо оптический диапазон? Должны же были смоделирвать. Она типа гладкая дохуя будет же, да? Падающее говно на фоне свечения будет незаметно? Тектоники и внешних всяких особенностей-фактур нет?
Аноним 23/10/21 Суб 06:33:00 #312 №687366 
>>687181
У нас и на сраной земляшке чугуний с люминем есть, зачем нам космических психов возить?
Аноним 23/10/21 Суб 06:34:31 #313 №687367 
image.png
>>687229
Скотт "Наше Всё" Мэнли.
https://www.youtube.com/channel/UCxzC4EngIsMrPmbm6Nxvb-A
Аноним 23/10/21 Суб 12:22:41 #314 №687398 
>>687367
Это афиша к "сквозь горизонт"?
Аноним 23/10/21 Суб 12:25:16 #315 №687399 
>>687365
>Она типа гладкая дохуя будет же, да?
Гладкий 10-километровый шарик, да.
Аноним 23/10/21 Суб 13:00:30 #316 №687408 
image.png
А кто-нибудь моделировал детально столкновения нейтронных звезд? Чтоб там кора трескалась, приливные силы друг дружку хуячили, разогревы-смятия, вываливающиеся спагетти и страпельки, сублимация массивных ядер, и какой-то момент когда это вдруг черная дыра становится?
Аноним 23/10/21 Суб 13:45:11 #317 №687410 
>>687408
>кора
Аноним 23/10/21 Суб 14:19:43 #318 №687413 
image.png
>>687410
Ты, судя по всему, не знаешь о коре нейтронной звезды.
Аноним 23/10/21 Суб 14:43:07 #319 №687414 
>>687413
Ты часто веришь картинкам из интернета, и употребляя слово в его первоначальном значении в контексте обсуждения нейтронной звезды?
Аноним 23/10/21 Суб 14:50:53 #320 №687415 
>>687414
Нахуй ты упорствуешь в своей тупости, еблан? Обосрался с незнанием существования коры - обтекай молча.
Аноним 23/10/21 Суб 15:26:24 #321 №687421 
>>687408
>кора трескалась
Лол, она жидкая. (При таких нагрузках) Макароны ты один хрен не увидишь, они тонкие и рвутся.
Там как сплошную каплю моделируют, ЕМНИП.
Аноним 23/10/21 Суб 15:49:14 #322 №687423 
>>687410
https://www.ncbi.nlm.nih.gov/pmc/articles/PMC5255077
Аноним 23/10/21 Суб 15:49:24 #323 №687424 
Откуда взялся большой взрыв? Являются ли квантовые флюктуации возможной причиной рождения вселенной, или это мемный аргумент учёных чтобы от них отъебались и не задавали глупых вопросов на которые они не могут ответить? "Ну тип кароч там флюктуации какие-то вот гуглите чё это такое и идите нахуй".
Аноним 23/10/21 Суб 15:56:44 #324 №687425 
>>687415
Быстро порвалась, слишком очевидно
Продолжай штудировать википедию и срать здесь своими вопросами уровня 3А класса
Аноним 23/10/21 Суб 15:57:43 #325 №687426 
>МАМ НУ В НЕЙТРОННЫХ ЗВЕЗДАХ ЖЕ КОРА, Я УМНЫЙ ДА? СЕЙЧАС СПРОШУ КАК ОНА СТАЛКИВАЕТСЯ!!!!

Пиздец. 2021 год на дворе. Интернет. Неограниченные объемы данных
Аноним 23/10/21 Суб 17:22:59 #326 №687432 
>>687424
>Откуда взялся большой взрыв?
Ага, я тебе расскажу, а ты тут же побежишь в нобелевский комитет за премией, держи карманы шире.
Аноним 23/10/21 Суб 17:41:18 #327 №687434 
Отрывок из фантастического романа.
Вам что-нибудь известно о новой звезде в созвездии Орла?

Дэв откинулся назад и начал просматривать банки данных своей оперативной памяти. Да, ему удалось найти кое-что по этому вопросу, отрывочные сведения, полученные много лет назад, ещё в те времена, когда он читал обо всём, что касалось звёзд. Он знал, что на земном небосклоне существует область в созвездии Орла, где одно время было зафиксировано необычно много новых, т.е. взрывающихся звезд. В течение одного сорокалетнего периода в первой половине XX века двадцать пять процентов всех новых звезд, наблюдаемых с Земли, появились в районе, равном одной части процента всего неба; только в одном 1936 году там появились сразу две новые, а Новая звезда Орла 1918 года стала самой яркой из всех звезд, зарегистрированных на протяжении трёх столетий и затмила все остальные звезды небосвода, кроме Сириуса.

Это правда или автор сочинил. В интернете ни чего кроме собственно новой 1918 не находится.
Аноним 23/10/21 Суб 17:44:15 #328 №687435 
>>687432
Ну так ясное дело ничего конкретно доказанного нет. Просто интересно, есть ли какие то теории которые хоть немного претендуют на достоверность, или ничего более чем "ну хуй его знает" на данный момент нет.
Аноним 23/10/21 Суб 20:07:31 #329 №687446 
>>687424
>Откуда взялся большой взрыв?
Большой взрыв был обеспечен стадией инфляции, которая была до него.
Аноним 24/10/21 Вск 01:08:16 #330 №687468 
>>687425
Ебальник закрой, даун тупорылый. Такому тупорылому быдлу упорствующему в своем невежестве не место в интернете. Чтоб тебя собутыльник зарезал, мразь ебаная, бесполезный биомусор.
Аноним 24/10/21 Вск 01:08:47 #331 №687469 
>>687426
>>687468
Аноним 24/10/21 Вск 03:17:00 #332 №687472 
>>687435
Официальный ответ навуки
>Ну может там ебнуло что-то
>Хуй знает
>А может боженька создал
>Ну хуй знает в общем
Аноним 24/10/21 Вск 03:28:44 #333 №687474 
>>687472
Всяко лучше, чем раз и навсегда выдумать окончательный неправильный ответ и прекратить исследования, поставив крест на надежде узнать правильный.
Аноним 24/10/21 Вск 12:52:53 #334 №687489 
>>687468
Если ты так часто рвешься, как же ты попу зашиваешь, дауненок?
Аноним 24/10/21 Вск 14:18:18 #335 №687509 
Чем ограничевается максимальное количество модулей на МКС? Если туда присобачить 20,30,40,50 модулей, что будет?
Аноним 24/10/21 Вск 14:42:36 #336 №687520 
image.png
>>687468
>Такому тупорылому быдлу упорствующему в своем
Аноним 24/10/21 Вск 16:06:52 #337 №687524 
14387318695850.png
>>687509
Раскачиваться начнёт, придётся моды с растяжками ставить.
Аноним 24/10/21 Вск 17:33:50 #338 №687531 
>>687509
Неиронически вот этот прав >>687524 - сложная механика всей этой колышущейся ебалы. Вообще, большие тонкие конструкции в микрогравитации это нетривиальная хуйня, особенно относится к солнечным панелям, а уж если аппарату надо поворачиваться, буститься, если на нём есть какие-то внутренние источники вибрации.

А вообще факторов овердохуя, начиная от приливного захвата (в который попадёт такая станция) до требований по энергетике, термодинамике, обслуживанию. даже космонавты заблудятся, блеать
Аноним 24/10/21 Вск 17:34:54 #339 №687532 
>>687509
Чем больше модулей, тем больше модов колебаний возникает. Из-за отсутствие опоры вибрации будут рассеиваться в каких-то точках, взывая их деформацию и в конечном итоге разрушение.
Впрочем можно запилить внешние фермы и ним жестко закреплять модули. Но тут уже другой целый вагон проблем булет.
Другой фактор это центр масс станции. Момент инерции станции довольно быстро растет, а вместе с ним затраты на ориентирование станции. А если станция достаточна большая, то уже начинают сказываться приливные силы. Если оси инерции будут сильно различными, то станция будет постоянно закручивать в лихом брекдансе.
Аноним 24/10/21 Вск 21:16:27 #340 №687560 
Вот когда пишут про "межзвёздную пыль" или "межзвёздный газ" - это имеется в иду прям пыль-пыль, как у меня под кроватью или же скопления каких-то более крупных объектов, типа Фобоса-Луны-Меркурия?

"Юпитер газовый гигант" - это прям облако сжиженного газа(метана, например) или скопление камней, которые парят таким роем и с масштабах солнечной системы всё равно что газ? Насколько понимаю, совершить посадку на поверхность Юпитера в любом случае некуда? Нет постоянной твёрдой основы?
Аноним 24/10/21 Вск 21:21:18 #341 №687563 
>>687560
>это имеется в иду прям пыль-пыль
Да.

>"Юпитер газовый гигант" - это прям облако сжиженного газа
Не сжиженного, а обычного. В основном водорода. Ну, с глубиной с ростом давления он постепенно переходит в жидкую и твёрдую фазу, но поверхности нет.
Аноним 24/10/21 Вск 23:12:18 #342 №687576 
>>687560
Межзвёздную пыль это любые мелкие частиц меньше миллиметра, но в основном это частички меньше микрона, по больше части состоящие из льда, силикатов или железа. Пыль под кроватью это органические полимеры, скомканные вокруг мелких частиц глины. Такая пыль естественно в космосе не встречается.
С межзвёздным газом есть путаница. Дело в том, что газ из нейтральных атомов и простых молекул видны только тогда, когда у их достаточная плотность(тогда уже говорим о гигантских облаках газа) и есть рядом мощный подсвечивающий источник. В остальных случаях они остаются невидимы для наблюдений. Однако к межзвёздному газу относят еще и ультраразряженную плазму, состоящие из протонов, электронов и ядер гелия. Эта плазма светится сама по себе(в рентгене) и вдобавок поляризует радиоизлучение. Из-за такой хорошей заметности плазму называют межзвёздный газ, хотя она является малым компонентом межзвёздной среды.

Да, Юпитер это большое облако газа. Из-за высокого давления газ не имеет четкой границы жидкой фазы и плавно переходит в сверхкритический флюид. Это как облако, только оно все гуще и гуще при погружении. Камни на Юпитере есть, но они считай в самых глубокий недрах, почти в центре.
Аноним 24/10/21 Вск 23:16:10 #343 №687577 
>>687563
>>687576
Во. Популярно и понятно.
Спасибо.
Вам бы учебники писать.
Аноним 25/10/21 Пнд 03:17:42 #344 №687583 
>>687576
На мой взгляд профана все верно сказано, единственное дополню,что плазма не только в рентгене светит, но и в радиоволнах.
Аноним 25/10/21 Пнд 14:13:57 #345 №687627 
>>687576
Вот чисто интересно, если хотя бы часть этого каменного ядра у Юпитера изъять и доставить на Землю - ученые найдут там что нибудь интересное, или это будут типичные силикаты?
Аноним 25/10/21 Пнд 15:45:02 #346 №687641 
>>687627
Там не может быть просто силикатов. Из-за охуевшего давления там немного другая химия. Но как только ты давление снимешь - они тут же разложатся на неинтересные силикаты и неинтересные добавочные элементы
Аноним 25/10/21 Пнд 17:14:23 #347 №687656 
>>687627
>>687641
Современная кристаллография предсказывает, что при таких давлениях могут проходить невозможные при обычных условиях реакции и образовываться всякая дичь вроде NaCl7 или Mg3O2. Поэтому скорее всего там что-то сильно экзотическое.
>Но как только ты давление снимешь - они тут же разложатся на неинтересные силикаты и неинтересные добавочные элементы
Тоже не 100% обязательно, некоторые из таких соединений должны быть метастабильны.
Аноним 25/10/21 Пнд 17:26:49 #348 №687658 
>>687656
>>687641
А если в лабораторных условиях к обычным земным силикатам применить условия сверхвысоких давлений?
Аноним 25/10/21 Пнд 19:08:31 #349 №687686 
>>687658
Такие давления пока не удаётся получить экспериментально даже близко. Алмазные наковальни это очень капризный метод наподобие поисков правильной точки в довоенных кристаллических детекторах. Даже те рекордные 770 гигапаскаль это "раз на раз не приходится" и были сделаны по сути ради рекорда. А это всего лишь небольшая доля от нескольких тысяч гигапаскаль в ядре Юпитера. Типичные цифры это пара сотен ГПа, причём там между ними мизерное количество материала, к которому применяют разные хитровыебанные косвенные методы чтобы доказать что получилось именно то что получилось, а другие челы доказывают что получилось совсем другое.

В общем с экспериментальной физикой высоких давлений пока полный цирк, а настолько высоких давлений чтобы как в ядре Юпа, и не могут получить даже.
Аноним 25/10/21 Пнд 23:25:13 #350 №687749 
>>687656
>могут быть метастабильны
Поправил тебя. Вся эта химическая дичь происходит из-за деформаций электронных оболочек и вероятность того, что после снятия давления электронам будет так удобно в скукоженном положении, крайне мала. Алсо, даже метастабильные формы обычных земных веществ имеют пределы устойчивости, вне которых быстро кровькишкираспидорасило переходят в стабильные
Аноним 27/10/21 Срд 18:00:37 #351 №688040 
Почему продукты в глубокой заморозке все равно портятся? А если они в герметичной упаковке будут хранится на орбите, в таком случае тоже испортятся? или после разморозки разницы не будет?
Аноним 27/10/21 Срд 19:03:27 #352 №688051 
>>688040
При заморозке ты только замедляешь процессы разложения. Совсем их остановить можно было бы, наверное, заморозив до абсолютного нуля, но до абсолютного нуля заморозить невозможно. Так что они в любом случае будут портиться.
Аноним 27/10/21 Срд 19:17:45 #353 №688052 
>>688040
Это какие продукты портятся при глубокой заморозке, и за сколько времени?
>>688051
Разлагают жратву микроорганизмы. Ниже нуля никакие микроорганизмы не работают.
Аноним 27/10/21 Срд 20:10:10 #354 №688062 
>>688052
Биополимеры гидрализуются сами по себе даже при очень низких температурах. При гидролизе вылезает всякое побочное говно, которое не очень желательно в еде.
Аноним 27/10/21 Срд 20:38:21 #355 №688068 
>>688062
Хуй знает, и сколько будет портиться сибирский мороженый суп на верёвочке? Даже консервы могут десятилетиями храниться (через несколько лет становясь всё менее съедобными, но есть их можно), а замороженная еда вообще будет столетиями пригодна в еду небось. Оригинальный же вопрос "продукты в заморозке портятся" мне вообще непонятен, т.к. что-то не верится без реальных цифр что что-то может значимо испортиться в виде ледышки, за осмысленный срок.
Аноним 27/10/21 Срд 20:44:23 #356 №688069 
>>688068
По-моему, у Солженицына было — в какой-то газете он встретил заметку, что в мерзлоте нашли мамонта, и тот настолько хорошо сохранился, что его там же сожрали. И Солженицын понял, что это были з/к, потому что голод для них оказался более важным фактором, чем палеонтологическая ценность.
Аноним 27/10/21 Срд 21:36:23 #357 №688073 
Чел, все продукты в морозилке портятся вообще то, а если теплее чем -18 еще и бактерии активны могут быть
Аноним 27/10/21 Срд 22:03:00 #358 №688074 
16267858747230.jpg
Были ли какие-нибудь новые открытия связанные с Гравитацией? Люди же до сих пор не знают, что такое Гравитация? Или уже поняли?
Аноним 27/10/21 Срд 22:34:42 #359 №688077 
Представим планету с более плотной атмосферой. По идее звук там должен распространяться на большем расстоянии, чем на Земле?
Учитывая что она будет необитаема (полная тишина), и там может какое то время находится человек такой вопрос:

Если он пукнет, звук может распространится на километры или нет?
Аноним 27/10/21 Срд 23:18:33 #360 №688080 
>>688074
>Люди же до сих пор не знают, что такое Гравитация?
Знают уже больше ста лет, с добрым утром. Гравитация - чисто геометрический эффект, искривление пространства массой тел.
Аноним 28/10/21 Чтв 00:04:08 #361 №688082 
>>688080
Если материя это энергия, а энергия это материя. То что есть колебания гравитации? Как их искусственно вызвать?
Аноним 28/10/21 Чтв 00:56:52 #362 №688084 
>>688082
Гравитация - это искривление пространства массой тел. Различные тела своей массой оказывают влияние на саму ткань пространства, деформируя ее. Чтобы получилась гравитация, тебе просто нужна масса.
Аноним 28/10/21 Чтв 03:10:22 #363 №688090 
>>688077
> Учитывая что она будет необитаема (полная тишина)
Не будет там тишины. Ветра дуют везде, даже на Венере, даже на Титане.
Аноним 28/10/21 Чтв 03:21:38 #364 №688091 
>>688084
>Гравитация - это искривление пространства массой тел.

А что такое "масса"?
Аноним 28/10/21 Чтв 04:25:33 #365 №688093 
Красное смещение.
Из-за расширения вселенной длина волны света, испущенного в миллиардах световых лет отсюда, увеличивается.

Но при этом фотоны теряют энергию, потому что синий свет содержит больше энергии, чем красный.
Внимание, вопрос: куда девается энергия?
Аноним 28/10/21 Чтв 04:47:43 #366 №688094 
>>688074
Пару лет назад подтвердили существование гравитационных волн.
Аноним 28/10/21 Чтв 04:51:57 #367 №688095 
>>688093
> Внимание, вопрос: куда девается энергия?
Пути господни неисповедимы.
Аноним 28/10/21 Чтв 09:21:03 #368 №688109 
>>688080
>искривление пространства массой тел
Если на Земле создадут большую плотную сферу, то она начнёт притягивать к себе объекты? Если из-за воздействия более массивного тела мы не заметим такого эффекта, тогда почему присутствует гравитация от Земли, при воздействии Солнца на Землю?
Аноним 28/10/21 Чтв 10:02:37 #369 №688111 
>>688109
Абсолютно все притягивает к себе объекты.
Даже ты.
Аноним 28/10/21 Чтв 10:45:02 #370 №688115 
>>688077
А может ты и не сможешь пукнуть там, а чтобы пукнуть, нужно будет жопу порвать.
Аноним 28/10/21 Чтв 10:46:03 #371 №688116 
>>688093
На расширение вселенной, очевидно же! Думал, она просто так расширяется?
Аноним 28/10/21 Чтв 11:12:11 #372 №688117 
1231242791816.jpg
>>688111
Моя коробочка с фотонами не согласна.
Аноним 28/10/21 Чтв 11:28:05 #373 №688118 
>>688093
>Из-за расширения вселенной длина волны света, испущенного в миллиардах световых лет отсюда, увеличивается.

Это лишь одна из гипотезы. Никто точно не знает причину красного смещения.
Аноним 28/10/21 Чтв 11:32:28 #374 №688120 
Дайте ссылку на теорию и как она называется.
Читал где-то, что через миллиарды лет звезда кончатся и останутся только черные дыры. Они со временем втянут в себя всю остальную материю, а потом в силу каких-то процессов в ЧД и между ними возникнет обмен информацией и однажды они станут чем-то вроде частей или нейронов огромного мозга, в котором из-за размеров одна мысль будет проходить тысячи лет, но мысли там будут. И вся наша белковая суета и даже звёзды это просто прелюдия к настоящей жизни.
Анон знает кто это придумал?
Аноним 28/10/21 Чтв 11:46:01 #375 №688124 
>>688120
>но мысли там будут
Какие мысли?, это просто сравнение было c промежутком времени, это из ТАЙМЛАПС БУДУЩЕГО, черные дыры испаряться, а потом протоны и вселенная типа продолжит вечно расширяться
Аноним 28/10/21 Чтв 11:54:55 #376 №688127 
>>688120
>Анон знает кто это придумал?
Просыпается как-то ученный с мировым именем с стокгольмской канаве, голова гудит, что было вчера не помнит (но помнит что вроде не Эйнштейн), а в кармане обрывок салфетки с текстом "ГЕНИАЛЬНАЯ ИДЕЯ: через миллиарды лет звезда кончатся..." в конце еще что-то про банан.
Аноним 28/10/21 Чтв 11:59:33 #377 №688129 
>>688124
Нет, я именно про зарождение сознания при взаимодействии черных дыр между собой.
Аноним 28/10/21 Чтв 13:00:06 #378 №688140 
>>688129
Какое сознание лол?
Симуляция того что ты называешь сознанием это один из способов выживания органики и не более того, и это не то к чему стремится материя сама по себе.
Аноним 28/10/21 Чтв 13:10:46 #379 №688141 
>>688117
Фотоны содержат энергию и тоже искривляют пространство
Аноним 28/10/21 Чтв 14:21:47 #380 №688149 
>>688117
Согласна. А если в неё напихать достаточно фотонов, она схлопнется в кугельблиц.
Аноним 28/10/21 Чтв 15:04:15 #381 №688154 
>>688109
Гравитацией обладает любое тело с массой.
Аноним 28/10/21 Чтв 15:08:47 #382 №688157 
>>688093
Из-за расширения пространства они и теряют энергию, прилетая покрасневшими. Они летят по растягивающемуся пространству, которое уносит их источник назад от наблюдателя.
Аноним 28/10/21 Чтв 15:14:47 #383 №688161 
>>688157
Ты вопрос-то попытайся понять.
Аноним 28/10/21 Чтв 15:19:36 #384 №688162 
>>688161
>пук
Аноним 28/10/21 Чтв 15:22:25 #385 №688163 
>>688157
Из-за расширения пространства они прилетают реже, чем испускались. Поэтому цвет и красный.
Конкретный фотон не имеет цвета.
Аноним 28/10/21 Чтв 15:28:05 #386 №688164 
>>688141
Пруфы будут, или ты в формулу циферки подставил?
Аноним 28/10/21 Чтв 16:20:14 #387 №688167 
>>688154
>Гравитацией обладает любое тело с массой
Тогда почему мы к горе не притягиваемся? Или к какому-нибудь высотному зданию.
Аноним 28/10/21 Чтв 16:27:32 #388 №688168 
>>688167
Притягиваемся, просто к горе мы притягиваемся в 100500 раз слабее чем к матушке-земле.
Аноним 28/10/21 Чтв 17:39:35 #389 №688172 
>>688167
Ещё как притягиваемся, первый эксперимент по измерению массы Земли как раз и заключался в проверке, насколько сильнее грузик притягивается к Земле, чем к горе.

https://en.wikipedia.org/wiki/Schiehallion_experiment
Аноним 28/10/21 Чтв 17:46:24 #390 №688174 
>>688093
Никуда, закон сохранения энергии работает только для плоского неподвижного пространства.

>>688163
Ты ебанат просто. Фотонов прилетает ровно столько же, но каждый конкретный фотон имеет длину волны, которая растягивается вместе с пространством. Фотоны реликтового излучения, например, растянуты в ~1100 раз и несут в 1100 раз меньше энергии, но количество их осталось ровно таким же.
Аноним 28/10/21 Чтв 18:00:52 #391 №688177 
>>688174
Расскажи как из удалённого источника, скажем, на 800 ТГц, получить за 10-12 сек те же 800 фотонов, но на сдвинутой до 200 ТГц частоте. Не понадобится ли тебе для этого 4 x 10-12 секунд?
Аноним 28/10/21 Чтв 18:06:43 #392 №688178 
>>688174
> Никуда, закон сохранения энергии работает только для плоского неподвижного пространства.
То есть, это можно и в обратную сторону провернуть, получая энергию, сжимая пространство?
Аноним 28/10/21 Чтв 18:09:24 #393 №688179 
Можно ли взвесить луну, если на ее поверхности положить весы обратной стороной и встать на них?:
Аноним 28/10/21 Чтв 18:11:18 #394 №688180 
>>688174
А если источник света движется на нас и свет приходит смещенным в синюю сторону, откуда появляется дополнительная энергия?
Аноним 28/10/21 Чтв 18:11:59 #395 №688181 
>>688177
По-моему, тебе стоит посетить школу ещё раз. Частота колебаний поля не имеет вообще никакого отношения к частоте испускания фотонов. В розетке частота 50 герц, но это не значит, что из неё 50 раз в секунду вываливается и втягивается гигантский фотон. Вайфай-роутер, микроволновка и спутники GPS работают на примерно одной частоте, но это не значит, что они испускают одинаковое количество фотонов.
Аноним 28/10/21 Чтв 18:14:21 #396 №688182 
>>688181
Роутер и не выпускает фотоны, ты учебник по физике скурил или что?
Аноним 28/10/21 Чтв 18:18:10 #397 №688183 
>>688182
А, всё понятно, дальше продолжать нет смысла.
Аноним 28/10/21 Чтв 18:20:50 #398 №688184 
>>688183
Действительно, иди ради приличия хотя бы видосы на ютубе посмотри...
Аноним 28/10/21 Чтв 18:21:50 #399 №688185 
Какие фотоны вообще. Коробку достань от вайфая и почитай он испускает килобайты, какие фотоны, это торшер по твоему?
Аноним 28/10/21 Чтв 18:25:04 #400 №688186 
1.png
2.png
>>688178
Так это и происходит при коллапсе звезд или молекулярных облаков. Они нагреваются за счет гравитационной энергии, которая "выделяется" при усиление кривизны. Правда это аналогия верна только при относительно слабых полях, когда можно обходится Ньютоновским приближением. В общем случае из-за принципа эквивалентности есть неоднозначности, которая пидорасить все в произвольной системе отсчета.

И вообще связь энергии-импульса и кривизны эта вся суть уравнения Эйнштейна. В более общем виде можно выразить через действие Гильберта-Эйнштейна.
Аноним 28/10/21 Чтв 18:26:45 #401 №688187 
>>688181
Лол, обосрался с гигантского фотона в розетке. Но всё же, вот есть постоянная Планка, которую умножают на частоту, и получают типа квант энергии. Точнее, энергию кванта, похуй. В моём примере выше я по недомыслию предположил, что фотоны вылетают из источника по одному за волну, оче слабый источник. Но ведь тогда на каком-то этапе излучение будет сдвинуто так, что на приём одного периода волны (просто чтобы понять, что это именно фотон прилетел) потребуется времени больше чем на испускание нескольких фотонов более высокой частоты.
Аноним 28/10/21 Чтв 18:32:47 #402 №688188 
>>688187
> Лол, обосрался с гигантского фотона
Ты поосторожнее с такими заявлениями, конденсат Бозе-Эйнштейна не поймет
Аноним 28/10/21 Чтв 18:38:00 #403 №688189 
>>688167
Лол, притягиваемся. Только этот эффект очень мал по сравнению с притяжением Земли, потому что разница масс между горами и Землей огромна.
Аноним 28/10/21 Чтв 18:41:41 #404 №688190 
>>688163
Цвет красный, потому что они летят по растягивающемуся пространству из тех областей пространства, которые в результате расширения отдаляются от наблюдателя. В этом вообще суть эффекта космологического красного смещения.
Аноним 28/10/21 Чтв 18:42:44 #405 №688191 
fa0afd7c03be45bc80d1425267cf118e.jpg
>>688188
Похуй, просушим кварко-глюонной плазмой.
Аноним 28/10/21 Чтв 18:43:17 #406 №688192 
>>688185
Долбоебина, роутер испускает радиоизлучение. Радиоизлучение - это поток фотонов. Поссал тебе на ебало.
Аноним 28/10/21 Чтв 18:43:48 #407 №688193 
>>688190
Охуенно пояснил, сразу понятно стало.
Аноним 28/10/21 Чтв 18:46:39 #408 №688194 
Мне одному идея "переносчиков взаимодействия" кажется несусветной тупостью? "Ужасное дальнодействие" и т.п. Типа для взаимодействия материи непременно нужны какие-то блять переносчики.
А переносчикам почему не нужны переносчики переносчиков?
Ведь это так и получается, что фотоны в электромагнитном взаимодействии самым что ни на есть образом дальнодействуют друг с другом.

Да и сам термин "взаимодействие" слишком тупой и устаревший. Это иллюстрирует даже тот факт, что слабое и электромагнитное взаимодействие оказалось как бы одним и тем же. Как бы, потому что сам термин - тупое говно тупого говна и происходящие в реальности процессы нихуя в него не вписываются.
Аноним 28/10/21 Чтв 18:48:34 #409 №688195 
>>688190
Где энергия, Лебовски? Звезда испустила 10 джоулей энергии, а пришло 2. Кто спиздил остальную энергию?
Аноним 28/10/21 Чтв 18:49:39 #410 №688196 
>>688185
Испустил несколько байт тебе за щёку, проверяй.
Аноним 28/10/21 Чтв 18:55:20 #411 №688197 
>>688192
АХАХХАХАХАХАХА
Ебать ты отбитый
Ну ка давай пруфы что радиоизлучение это фотоны
У меня лично ройтер килобайты испускает с двачем, где я могу таким тупичам как ты это объяснить
Аноним 28/10/21 Чтв 18:56:14 #412 №688198 
>>688193
>пук
>>688195
Потерялась в ходе движения в расширяющемся пространстве.
Аноним 28/10/21 Чтв 18:58:35 #413 №688199 
>>688197
Дегенерат, любое электромагнитное излучение представляет собой поток фотонов. Это выяснил еще Эйнштейн в начале прошлого века, объяснив фотоэффект, за что и получил нобелевку. Радиоизлучение - вид электромагнитного излучения. А теперь соси собачий хуй.
Аноним 28/10/21 Чтв 18:59:28 #414 №688200 
>>688197
см. >>688196
Аноним 28/10/21 Чтв 19:00:14 #415 №688201 
>>688194
Долбоеб, нахуй ты высрал этот чудесный поток сознания? Что сказать-то хотел?
Аноним 28/10/21 Чтв 19:02:37 #416 №688202 
>>688199
ТАК ГДЕ ДОКАЗАТЕЛЬСТВА ЭТОМУ
Почему никто из вас не даст мне ссылки на пруфы? Почему я должен верить на слово анонимам из двача сидящим с роутерами испукающими "фотоны"
Аноним 28/10/21 Чтв 19:05:14 #417 №688203 
>>688202
>ТАК ГДЕ ДОКАЗАТЕЛЬСТВА ЭТОМУ
У мамши твоей в пизде, я их туда хуем забил.
Аноним 28/10/21 Чтв 19:06:12 #418 №688204 
>>688198
> Потерялась
Как?
Аноним 28/10/21 Чтв 19:06:47 #419 №688205 

Собственно все что стоит знать об фанатиках которые здесь засели >>688203

Просишь ссылки на исследования, доказательства - поток оскорблений.
Аноним 28/10/21 Чтв 19:07:36 #420 №688206 
Вы еще блять скажите что я попкорн в микроволновке фотонами разогреваю, вы вообще что ли
Аноним 28/10/21 Чтв 19:12:19 #421 №688207 
>>688205
Чмо, ты задаешь вопросы, ответы на которые лежат в википедии. Тебя забанили в вики или ты настолько тупой, что не умеешь пользоваться поисковиком? Вбиваешь в гугл "фотоэффект" и наслаждаешься первой же ссылкой, ведущей на вики. Видишь, какой я добрый, даже подсказал тебе, тупому унтерменшу, какой именно запрос нужно вбивать в поисковик.
>>688204
Жопой об косяк.
>>688206
Именно фотонами ты его и разогреваешь, хуйло.
Аноним 28/10/21 Чтв 19:13:31 #422 №688208 
Ясно, хуесос трясется википедией, и даже ссылку не может дать.

ну нет у тебя пруфов так и скажи. Нахуй ты рвешься, манька? Свои фантазии одноклассникам в школе расскажешь.
Я вот никаких фотонов не вижу.
Сижу от роутера в метре.
Аноним 28/10/21 Чтв 19:15:55 #423 №688209 
>>688208
>Я вот никаких фотонов не вижу.
Я твоих мозгов тоже не вижу. Их точно нету. Уверен, если расколотить тебе нахуй черепушку, то там вместо мозгов будет вата какая-нибудь или собачий понос.
Аноним 28/10/21 Чтв 21:08:12 #424 №688214 
Каким способом эффективнее всего передавать энергию на растояние?
Когда мы лучом лазера светим на фотоэлемент.
Или когда мы ФАР направляем микроволновый луч на массив антенн, с которых электроэнергия снимается через выпрямитель?
Аноним 28/10/21 Чтв 21:39:51 #425 №688215 
>>688074
>что такое Гравитация
Люди до сих пор не выяснили, имя это или фамилия?
Аноним 28/10/21 Чтв 22:09:05 #426 №688217 
>>688209
>Я твоих мозгов тоже не вижу.
Потому что они в черепной коробке и фотоны от моих мозгов не испускаются.
А вот твои либо препарированы при рождении в банку - либо торчат наружу. Выбери сам. не забудь убрать светящий фотонами роутер
Аноним 28/10/21 Чтв 23:49:30 #427 №688237 
>>688217
>Потому что они в черепной коробке
Покажи мне, что они там.
>А вот твои либо препарированы при рождении в банку - либо торчат наружу.
Пидарас, спок. У тебя не получается в перефорс. Мозгов-то нет, лол.
Аноним 29/10/21 Птн 04:43:23 #428 №688264 
>>688215
Это фильм такой.
Аноним 29/10/21 Птн 06:56:37 #429 №688267 
>>688217
>>688237
Борьба была равна, боролись два говна.
А разгадка проста: два дебила - это сила.
Аноним 29/10/21 Птн 09:52:32 #430 №688283 
>>688202
https://www.booksite.ru/fulltext/1/001/008/060/311.htm
В зависимости от разности энергий состояний системы, между которыми происходит К. п., испускаются или поглощаются фотоны радиоизлучения, инфракрасного, видимого, ультрафиолетового, рентгеновского излучения, g-излучения
Аноним 29/10/21 Птн 09:57:18 #431 №688284 
>>688162
Ты хотел пукнуть, но обосрался. Бывает, иди падмойса.
Аноним 29/10/21 Птн 10:35:53 #432 №688287 
>>688195
Я же говорю, фотоны тормозятся о пространство, раздвигая его.
Аноним 29/10/21 Птн 10:40:14 #433 №688290 
>>688287
Мамаша твоя тормозится, раздвигая ноги.
Аноним 29/10/21 Птн 10:42:17 #434 №688291 
>>688290
Твоя даже не пытается тормозиться, перед всеми подряд раздвигает. Зато на сынке природа отыгралась, тот ещё тормоз.
Аноним 29/10/21 Птн 10:49:50 #435 №688293 
>>688290
>>688291
Школодебилы такие школодебилы.
Аноним 29/10/21 Птн 13:00:40 #436 №688321 
ELT4k-11-Night3cc.jpg
Поглядел я затраты на ELT в 1.1 лярда и на JWST в 10 лярдов и прикинул зачем так выебываться и запускать такой дорогой проект как jwst, когда можно вбухать все в наземный телескоп(построить еще больше зеркало) с которым и ремонт возможен и замена делателей, да и простоит он на целый век, чем Уэбб на лет 10 максимум рассчитанный, хули так?
Аноним 29/10/21 Птн 13:41:41 #437 №688326 
>>688321
Есть цели, которые с Земли не увидеть даже мощным телескопом. JSWT он же блядь спектроскопией во многом будет заниматься, а не просто фото делать. Хорошие фото конечно круто, но спектр будет засран земной атмосферой, а JSWT начали строить давно, и цена выросла непредсказуемо. В то время как ELT пока хуй знает когда достроят, может его цена тоже возрастёт
Аноним 29/10/21 Птн 14:52:45 #438 №688337 
>>688267
>пук
>>688284
Обосрался тебе в рот.
Аноним 29/10/21 Птн 15:17:17 #439 №688339 
Поясните, откуда такой дроч на шкалу цивилизаций? По факту это же просто фантазия, которая распространяет опыт последних нескольких веков человечества на все возможные или невозможные цивилизации. Моя кошка наверно тоже думает, что люди жрут Вискас ведрами.
Почему мы решили, что они ставят перед собой цели, требующие столько энергии? Почему её получение растет именно так — планета, заезда, другие звёзды, галактика? Почему они вообще берут ее со заезд? А может они вообще живут в астрале каком-нибудь или стоики дохуя и им ничего не надо, но при этом у них есть безграничные возможности? Да много чего может быть.
Я не наезжаю на Кардашева, это вполне себе нормальная теория. Но она периодически всплывает в видосах на Ютубе или в статьях на Вики как будто это какой-то постулат.
Аноним 29/10/21 Птн 15:19:43 #440 №688340 
Возможен ли в теории корабль из органики? С двигателем понятно будет сложно (хотя реактивные струи у насекомых никто не отменял) но например внешнее и внутреннее покрытие с комфортной для человека температурой, и составом воздуха?

Иными словами способна ли в принципе биоинженерия к созданию таких массивных механизмов?
Ну и второй вопрос аналогичный, только про скафандры из органики. Которые будут плотно прилегать к носителю и не пропускать излучений.
Аноним 29/10/21 Птн 15:25:59 #441 №688342 
>>688339
Так на эту шКАЛу дрочат только маняфантазеры. То, что надо сравнивать каждый аспект, им в безмозглую голову не приходит.
Видишь такого человекообразного, кинь в него кирпичом, может поумнеет.
Аноним 29/10/21 Птн 15:33:38 #442 №688345 
>>688340
Да. Только нужно время на развитие таких технологий, сам понимаешь, это тебе не нефть качать.
Аноним 29/10/21 Птн 15:34:50 #443 №688346 
>>688340
Очень вряд ли.
Органика слишком капризная и требовательная к условиям.
От УФ и протонов разрушаются, при перегреве или переохлаждении теряются нужные свойства.
Впрочем органика может быть использована для отельных узлов например как гасители механической энергии и щит от нейтронов. Углеродное волокно это органика?

Со скафандрами совсем беда. Дело в том, что реголит или любые силикаты, подверженные космическому выветриванию, становятся чрезвычайно химически активны по отношению к почти любой органике. Хотя если скафандр будет одноразовый и с возможностью его быстрой переработки, то это станет хорошим решением.
Аноним 29/10/21 Птн 15:49:18 #444 №688349 
>>688346
Наверное, он имеет в виду возможны ли организмы, способные выполнять функции скафандра и космического корабля.
Аноним 29/10/21 Птн 15:59:49 #445 №688350 
>>688339
>живут в астрале каком-нибудь
Астрала не существует.
Аноним 29/10/21 Птн 16:00:57 #446 №688352 
>>688349
Нет, я не про выращивание организмов в целом, а органических частей. Которые будут выполнять свои функции, а не быть огромной зверушкой.
Аноним 29/10/21 Птн 16:52:02 #447 №688360 
>>688352
Опять вряд ли, живые организмы дают много побочного говна.
Впрочем организмы можно включать в производственные циклы вроде перевода углекислого газа в графит или обогащение руды.

Однако будущее за 3D печатью или поатомной сборке корабля целиком.
Аноним 29/10/21 Птн 19:48:32 #448 №688426 
>>688350
Теория струн включает в себя 11 измерений. То что можешь воспринять ты 4-5 измерений. Возможно они чилят на остальных.
Аноним 29/10/21 Птн 19:49:05 #449 №688427 
1635526143884.jpg
1635526143890.jpg
1635526143910.jpg
Аноним 29/10/21 Птн 19:53:27 #450 №688429 
>>688426
Так остальные свернуты.
Аноним 29/10/21 Птн 21:36:22 #451 №688444 
>>688426
>Теория струн
Бесполезная хуита, которая не дает никаких проверяемых предсказаний, следовательно не соответствует критерию фальсифицируемости, а значит и находится вне сферы науки. Это как раз что-то на уровне астрала, торсионных полей и т.п. Мимо.
Аноним 29/10/21 Птн 21:40:56 #452 №688445 
Можно ли создать силовое поле с помощью левитации сверхпроводящих частиц?
Аноним 29/10/21 Птн 22:56:39 #453 №688471 
>>688445
В смысле силовое поле? Электромагнитное поле и так по определение силовое и любой движущийся заряд будет порождать электромагнитные поля.
Или ты про силовое поле из фантастики? Тогда в твоем случае нет. Единственное что более менее реализуемое в реальность это замкнутый поток ионов или плазмы в некоторой области, но в атмосфере эта хуита быстро рассеется.
Аноним 30/10/21 Суб 00:00:22 #454 №688479 
TheLexx002.jpg
>>688349
Lexx
Аноним 30/10/21 Суб 02:14:12 #455 №688489 
Почему ядро Земли так медленно остывает? Интуитивный ответ - эмиссия энергии через тепловое излучение неэффективна из-за отсутствия газа в космосе. Но ведь на Земле не так жарко с учётом светящего каждый день Солнца. Значит термоизоляция лежит где-то под землёй, верно?

Может ли запуститься термоядерный синтез в глубине очень большой планеты? Такой, чтобы давления хватило для поломки атомов и такой, чтобы эта минизвезда осталась в глубине планеты и дальше не росла?
Аноним 30/10/21 Суб 02:24:40 #456 №688490 
CarlSaganPlanetarySocietycropped.png
mind-blow-galaxy.gif
Почему быдло так дрочит на пикрилейтед? Что он такого изобрел, открыл, написал? Говно уровня Билла Ная и Нила Диграса Тайсона.

"Вау, чувак, ну там кароч космос такой огромный что прям аж крышу сносит ну и у нас типа голубая маленькая планета и всеж мы люди"

В чем я не прав?
Аноним 30/10/21 Суб 03:47:35 #457 №688492 
>>688489
препятствие для термоядерного синтеза внутри земли.
-недостаток топлива. водород и гелий поднимаются все выше. тяжелые элементы до железа могут термоядерносинтезировать вроде только в ОЧЕ БОЛШО ЗВЕЗДЕ. так как требуют особо много энергии.
-у ядра нет лицензии от росатома на работу с ядерными технологиями (хотя пример стабильной реакции деления урана внутри земной коры без человека есть в Габоне )

что мешает остыванию.
-считается что земля испускает инфракрасные лучи, но они якобы отражаются некоторыми атмосферными газами и возвращаются на землю. а для испускания видимого света и ультрафиолета, земля недостаточно энергетически насыщена. вот если поджечь..

это конечно не единственные причины. наверняка и температура в центре мала. и есть какие то ингибиторы термояда. и вокруг земли нет вещества вне атмосферы чтобы тепло отводить кроме как ик излучением
Аноним 30/10/21 Суб 03:51:58 #458 №688494 
>>688490
Ты даун а он один из величайших просветителей эпохи.
Всё.
Аноним 30/10/21 Суб 04:07:11 #459 №688499 
>>688489
>Почему ядро Земли так медленно остывает?
За счет радиоактивных элементов.
Аноним 30/10/21 Суб 04:12:03 #460 №688500 
>>688489
>эмиссия энергии через тепловое излучение неэффективна из-за отсутствия газа в космосе
Чё-то странная интуиция. Лампочка в вакууме тоже будет хуже светить из-за отсутствия приёмника света?
Аноним 30/10/21 Суб 04:16:13 #461 №688502 
>>688494
Это тот просветитель, который участвовал в сочинении послания вовне и создании табличек для Пионеров и прочих Вояджеров? Больше похоже на мракобесие на потеху публике. Не то чтобы это не нужно было, но то, что получилось, по исполнению полный пиздец.
Аноним 30/10/21 Суб 10:14:14 #462 №688539 
>>688490
Дай угадаю, ты Хокинга тоже хейтишь?
Аноним 30/10/21 Суб 10:38:14 #463 №688546 
>>685225 (OP)
Есть карта видимости авроры в ближайшее время? Коронарный выброс вроде бы был недавно.
Аноним 30/10/21 Суб 11:10:06 #464 №688550 
>>688489
>Почему ядро Земли так медленно остывает?
Потому что тепловая масса огромна, а термодинамический баланс (входящий/исходящий поток тепла) близок к нейтральному. Вот на сколько ватт на кв.м. Солнце греет - примерно столько же планета излучает в остальной космос. Кроме Солнца есть ещё ядерные реакции и приливный разогрев, правда вклад у них миллипиздрический.
>Может ли запуститься термоядерный синтез в глубине очень большой планеты?
Может, это называется коричневый карлик, и самый нижний порог для запуска синтеза это приблизительно 13 масс Юпитера.
Аноним 30/10/21 Суб 11:20:33 #465 №688552 
7b00ee1018dacde696cb343f7c62e60d.jpg
>>688340
Возможен. Более того, всю историю делали корабли из органики, пикрилейтед.

Насчёт плотно облегающих скафандров. Общий принцип, по которым они должны работать - примерно понятен: нужно "просто" механически давить на тело в нужных местах, компенсируя давление. Проблема в том, чтобы 1) нацепить такой скафандр и 2) сделать его эластичным. В насе пытаются делать нечто подобное, в час по чайной ложке. Результатов пока нет. Из идей - гидравлические линии, пропущенные через ткань, пьезо/магнетострикционные нити, и т.д и т.п.
Аноним 30/10/21 Суб 14:53:52 #466 №688583 
Магнитные бури влияют на самочувствие и здоровье человека?
Аноним 30/10/21 Суб 14:55:16 #467 №688584 
thumb-1920-377820.jpg
>>688479
Ты бы ещё ульи из звёздных врат вспомнил
Аноним 30/10/21 Суб 15:04:51 #468 №688586 
>>688583
Смотря какого.
Аноним 30/10/21 Суб 15:25:45 #469 №688594 
>>688500
Причём здесь свечение, если речь про тепловое инфракрасное излучение. Оно очень долго отводит энергию, если оно одно без конвекции. Именно поэтому в открытом космосе ты умрёшь от гипоксии, не от замерзания в ледышку.
Аноним 30/10/21 Суб 15:36:17 #470 №688597 
image.png
>>688594
О да, разница прямо-таки принципиальная.
Аноним 30/10/21 Суб 16:24:41 #471 №688606 
>>688500
Кстате, а по идее лампочка в космосе сможет и без стекла светить? Внуторь газ накачан чтоб не окислялась спираль ведь, значит вакуум норм?
Аноним 30/10/21 Суб 16:31:19 #472 №688607 
>>688606
Первые лампочки Ильича Эдиссона были заполнены вакуумом.
Аноним 30/10/21 Суб 16:47:20 #473 №688609 
>>688539
Хокинг учёный был как раз, а не открыватель рта на публику (и не только потому, что у него он физически не открывался).
Аноним 30/10/21 Суб 16:48:39 #474 №688610 
>>688607
А как это делали? Были специальные баллоны с сверхжатым вакуумом под давлением или как?
Аноним 30/10/21 Суб 16:53:28 #475 №688611 
хорошая шутеечка.mp4
>>688610
Аноним 30/10/21 Суб 17:01:22 #476 №688613 
image.png
image.png
>>688479
>>688584
Эти тоже. Но вообще было одно чтиво, где пришельцы именно выращивали органические корабли (включая мозговой отдел с контрольной медалью), управлять и находится в них было можно только с помощью экзоскелета- скафандра.
В нем тоже была контрольная медаль, которая имела связь с мозгом носителя. Таким образом они могли дистанционно управлять кораблем, как собственной частью тела.
Аноним 30/10/21 Суб 18:03:14 #477 №688622 
>>688610
Жидкий концентрат наливали
Аноним 30/10/21 Суб 22:44:09 #478 №688669 
>>688597
Ну да, видимое излучение не несёт ТЕПЛО, а речь именно о нём.
Аноним 30/10/21 Суб 23:16:30 #479 №688681 
>>688669
В смысле не несет? Солнце или лампочканакаливания разве не передают тепло через него?
Аноним 31/10/21 Вск 00:17:11 #480 №688693 
>>688681
Ну, справедливости ради стоит отметить, что и Солнце, и лампочка сильно фонят в ИК, по сравнению, например, со светодиодом.
Аноним 31/10/21 Вск 01:34:09 #481 №688705 
>>688681
Солнце вещает во всех диапазонах, от радио до гамма излучения. Светодиодная лампа вообще не нагревается, а разогретая каменная плита не светится.
Аноним 31/10/21 Вск 02:04:23 #482 №688709 
Немного нестандартный вопрос (не нашел более подходящего места)

Я ищу название породы камней. Это точно были камни естественного происхождения.
14-17 лет назад их возили тоннами (РФ)
Обычно перед тем как асфальтировать участки.

По цвету они были синеголубыми с белыми вкраплениями. По размеру - крупные куски, один с трудом умещался в кулаке.
Такой камень явно не был особо ценным и шел для того, чтобы разравнивнять дорогу. Или участок перед домом.

Вы меня спросите - мол а нахуя тебе это вообще и почему ты про это вспомнил.
Отвечу. В детстве мы искали самые редкие потому что у них были металлические прожилки серебряного/золотого оттенка.
Эти прожилки также имели металлический блеск.

Какой в этих камнях содержался металл - я так и не узнал. Именно поэтому ищу название.
Аноним 31/10/21 Вск 04:36:15 #483 №688719 
>>688489
>Может ли запуститься термоядерный синтез в глубине очень большой планеты?
Не может, т.к. тело, в недрах которого идут термоядерные реакции, называется звездой, а не планетой.
Аноним 31/10/21 Вск 04:56:53 #484 №688721 
>>688709
Не тут ищи. У тебя есть место и время. Делай запрос в отвественные места кто и когда и как делал такие-то работы. Откуда завозили породы ты тоже можешь выяснить.
На очевидный вопрос "че тебе делать нечего хули ты хуйню спрашиваешь" отвечаешь "пишу научную работу, если не можете ответить дайте кто ответит".
Мыло, созвоны, буквальная почта.
Тут не ленишься и находишь.
Аноним 31/10/21 Вск 10:49:58 #485 №688734 
>>688709
Возможно гранит с вкраплениями пирита и/или халькопирита.
Аноним 31/10/21 Вск 11:33:44 #486 №688740 
>>688734
Очень похоже на пирит, скорее всего он и был
Аноним 31/10/21 Вск 14:18:13 #487 №688772 
Что будет с черным карликом когда он потухнет? Как он будет выглядеть и из чего состоять? Если он будет холодным, то можно ли будет на нем высадиться и погулять? И что будет с ним дальше, через например ну пусть 1000 миллиардов лет? Рано или поздно он рассыплется в труху, не?
Аноним 31/10/21 Вск 14:29:38 #488 №688774 
>>688772
Вот как ты думаешь, сможешь ли ты приземлиться на черный карлик, если даже на Земле под ее поверхностью тебе будет ЖАРКО и ты элементарно сгоришь?
Аноним 31/10/21 Вск 14:31:09 #489 №688776 
>>688772
нихуя с ним не будет
разве что протоны распадутся или туннелирует в чд, но это пока сугубо теоретические процессы
Аноним 31/10/21 Вск 14:32:16 #490 №688777 
>>688772
>Что будет с черным карликом когда он потухнет?
Ты имел в виду, наверное, белый карлик, который потухнет и станет черным карликом. Нихуя с ним не будет. Будет болтаться овердохуя лет в космосе, пока не наткнется на какую-нибудь черную дыру, которая его сожрет.
>Как он будет выглядеть
Соответственно своему названию и будет выглядеть. Черный шарик размером с Землю, который нихуя не излучает.
>из чего состоять?
Из того же, из чего состоял, когда был белым карликом - из обычного вещества, т.е. из протонов и нейтронов.
>можно ли будет на нем высадиться и погулять?
Тебя распидарасит нахуй его гравитацией, т.к. это очень плотный объект. Так что нет, нельзя.
Аноним 31/10/21 Вск 15:24:50 #491 №688793 
>>688705
>Светодиодная лампа вообще не нагревается
Сразу видно человека, у которого не было закрытого плафона со светодиодными лампами, там лампы перегреваются и дохнут только так. У любой лампы около половины потребляемого электричества уходит в тепло.
Аноним 31/10/21 Вск 15:46:44 #492 №688807 
>>688793
Ну извините, все до чего додумались приматы за 2021 год это накаливать вольфрам
Аноним 31/10/21 Вск 16:22:22 #493 №688816 
>>688793
так это не светодиод, а евойный блок питания, если какую ленту потрогать то она почти комнатной температуры будет
Аноним 31/10/21 Вск 16:35:24 #494 №688819 
>>688816
Нет, мощные матрицы специально лепят на алюминиевый радиатор. Драйвер в этом случае обычно стоит отдельно на своей плате и со своим радиатором. Впрочем это не отменяет того, что при одинаковой светимости лампа накаливания греется сильнее.
Аноним 31/10/21 Вск 16:58:44 #495 №688831 
А насколько перспективно использовать на Земле и на околоземных станциях - органические лампы?
Аноним 31/10/21 Вск 17:04:41 #496 №688833 
>>688831
OLED не применяется для освещения из-за быстрого выгорания при большой яркости.
Аноним 31/10/21 Вск 17:52:45 #497 №688841 
Отличается ли невесомость на МКС от невесомости допустим на пути к Марсу?
Аноним 31/10/21 Вск 17:58:49 #498 №688843 
>>688841
Невесомость на МКС из за того что она находится в постоянном падении, там микрогравитация. По пути на Марс вообще не будет невесомости, так как там по любому запилят какой-нибудь противовес с помощью которого корабль будет крутиться как кусочек говна в унитазе.
Аноним 31/10/21 Вск 18:09:17 #499 №688848 
>>688772
>можно ли будет на нем высадиться и погулять?
Тебя распидорасит. Там же гравитация на поверхности ебовая.
Аноним 31/10/21 Вск 18:12:12 #500 №688850 
>>688843
А вот если не будет крутится, тогда состояние невесомости будет отличаться?
Аноним 31/10/21 Вск 18:18:09 #501 №688853 
>>688793
Именно. Плохое понимание этой темы налицо.

>>688807
Нет же. Куча разных физических способов фотоны генерить придумана.

>>688816
Нихуя подобного. КПД светодиодов где-то треть. Грубо говоря в идеальном случае 40% энергии становится светом. Остальное куда девается? В нагрев элемента.
Натриевые газоразрядные лампы более эффективны по преобразованию электричества в свет. Но у них свет желтый и срок жизни меньше. Как диоды подешевели - начали на них менять освещение потому что это не так дорого, зато свет белый а не желтый.
Аноним 31/10/21 Вск 18:19:29 #502 №688854 
>>688843
Сомневаюсь. Первый полёт будет флаговтыком и полетят туда абы как поскорее в 2050 годах. Можешь скринить.
Аноним 31/10/21 Вск 18:19:52 #503 №688855 
>>688841
на мкс 70% от g, на пути к марсу смотря будут по пути попукивать горючкой или нет
Аноним 31/10/21 Вск 18:26:40 #504 №688858 
>>688855
>70% от g
Чочо.
Аноним 31/10/21 Вск 18:33:18 #505 №688861 
>>688853
> КПД светодиодов где-то треть
это с каким люминофором? или примерно почуствовал?
Аноним 31/10/21 Вск 18:34:12 #506 №688862 
>>688861
Это примерно у всех, блядь, я загуглил.
Аноним 31/10/21 Вск 18:35:30 #507 №688864 
>>688858
на высоте мкс такое ускорение свободного падения
Аноним 31/10/21 Вск 18:37:12 #508 №688865 
>>688862
заскринь че ты там загуглил, у тебя первой строчкой в гугле вылезет 95%, хуле ты пиздишь
Аноним 31/10/21 Вск 18:38:14 #509 №688866 
image.png
>>688865
Аноним 31/10/21 Вск 18:42:54 #510 №688867 
>>688866
> led light source
> светодиодов
вот это самоприклад
Аноним 31/10/21 Вск 18:49:27 #511 №688871 
>>688867
Light emmiting diode по-твоему как-то по-другому переводится?
Аноним 31/10/21 Вск 19:17:53 #512 №688874 
>>688867
Ты сейчас сам себя приложил мета-комментарием что ли?
ЛЕД это светодиоды, если что.
Аноним 31/10/21 Вск 19:21:29 #513 №688875 
1014027206.jpg
>>688874
Led - это свинец, вообще-то
Аноним 31/10/21 Вск 19:26:16 #514 №688876 
image.png
>>688875
The reason they call it lead is the result of the same ignorance that resulted in calling the North American natives Indian. You know, those early explorers, when they found America, they thought they were in India so they call them Indian. Later British tried to fix it by eliminating natives, which they failed. So they took over India and tried to turn them into British, which they failed. Now we have to call the real Indians, East Indians, south Asian, Asian Indian, or whatever to avoid mixing mixing them with natives who were wrongfully accused of being Indian. ARGH! I wasn’t even involved!

The point is that people were originally using lead to draw. So when they discovered graphite they thought it does the same thing so it must also be an Indian.
Аноним 31/10/21 Вск 19:55:48 #515 №688884 
>>688875
Это сратшип выполняет флип манёвр над мехазилой?
Аноним 31/10/21 Вск 20:02:27 #516 №688887 
>>688874
Мета само приклад
Аноним 31/10/21 Вск 21:51:24 #517 №688936 
>>688793
>>688853
Так, признаю свою ошибку. Значит у меня давно засело такое заблуждение. В любом случае без обволакивающего газа теплопотери меньше. Свет не такое большое "окно" для отдачи энергии. А с конвекцией ещё газ таскает тепло туда-сюда.
Аноним 31/10/21 Вск 22:58:32 #518 №688952 
Объясните один вопрос.
Возможно ли в теории, в земных условиях (то есть находясь на Земле) - создать область с противовесом силе гравитации?
Размер области не важен, пусть хотя бы микрон.

Но это не должно делаться при помощи костылей.
Т.е. не при помощи отталкивания платформы от сгенерированных электромагнитных полей, или левитации сверхпроводников
Аноним 01/11/21 Пнд 01:36:44 #519 №688968 
>>688952
В центре земного шара вполне себе стабильная невесомость, например.
Аноним 01/11/21 Пнд 02:08:03 #520 №688972 
>>688968
Это тоже логический костыль. Понятное дело что в центре земли никакого оборудования для исследований как и людей в 21-23 веке не будет
Поэтому вопрос актуален для поверхности или той глубины где будет функционировать электроника/и может находится человек
Аноним 01/11/21 Пнд 02:20:06 #521 №688973 
Kate-Upton-Zero-Gravity-972968.jpeg
Kate-Upton-Zero-Gravity-972966.jpeg
>>688972
Кольцевая железная дорога вдоль экватора с поездом на первой космической скорости. Но этого тоже не будет. Так что пока максимум — это самолётики.
Аноним 01/11/21 Пнд 08:42:31 #522 №688992 
>>688968
>>688973
Оба варианта сильно хуже по качеству невесомости, чем орбита. (вибрации, неравномерность гравиполя и т.п.). На орбиту всяко проще запульнуть.

>>688972
При таких условиях - нет.
Аноним 01/11/21 Пнд 08:48:06 #523 №688994 
>>688843
На Марс лететь полгода максимум по гомановской траектории, поэтому нет особого смысла пилить центрифугу.

>>688613
С чтивом, дистационно управляемыми мозгом бионическими кораблями - это к научной фантастике.
Аноним 01/11/21 Пнд 11:51:01 #524 №689015 
>>688502
Чмоня с харкача (ты) сделала бы лудше, конечно же.
Аноним 01/11/21 Пнд 13:09:03 #525 №689024 
>>688994
>управляемыми мозгом

Это уже к фантастике не относится. Сейчас есть люди с имплантами вполне способными набирать текст на клаве.
Аноним 01/11/21 Пнд 14:17:37 #526 №689034 
>>689015
>лудше
Лучше, чем ты, сделать немудрено
Аноним 01/11/21 Пнд 14:29:03 #527 №689035 
>>689034
Ну у тебя точно 85 icq
Аноним 01/11/21 Пнд 14:54:40 #528 №689036 
На Землю летит железнокаменный астероид с массой несколько сотен тонн.
У вас есть 20 часов и ресурсы двух крупных стран.
Ваши действия?
Аноним 01/11/21 Пнд 15:14:31 #529 №689037 
>>689036
Выкладываю на двачи пак с процессорами.
Аноним 01/11/21 Пнд 15:29:20 #530 №689038 
image.png
>>689035
Спасибо!
Аноним 01/11/21 Пнд 15:41:39 #531 №689040 
>>689036
Эвакуировать население и укрываться в складках местности
Аноним 01/11/21 Пнд 15:42:59 #532 №689041 
>>689040
В чьих складках?
Аноним 01/11/21 Пнд 15:44:56 #533 №689042 
image.png
>>689036
> несколько сотен тонн
Аааще похуй абсолютно. Максимум где-то стёкла повыбивает.
Аноним 01/11/21 Пнд 16:45:22 #534 №689049 
>>689037
Времени уже мало, мы ждем!
Аноним 01/11/21 Пнд 18:39:19 #535 №689070 
У вас уже на несколько часов меньше
Спасайте Землю от астероида, я не шутил....
Аноним 01/11/21 Пнд 18:54:42 #536 №689072 
>>689070
Ты ж написал что несколько сотен тонн. Вообще насрать на такой астероид, он нихуя не сделает.
Аноним 01/11/21 Пнд 19:16:55 #537 №689073 
>>689072
Ты уверен?
Аноним 01/11/21 Пнд 19:30:04 #538 №689075 
>>689073
Максимум стекла повыбивает, земле насрать вообще.
Аноним 02/11/21 Втр 02:39:26 #539 №689128 
Ну и сидите все с выбитыми стеклами!
Аноним 02/11/21 Втр 09:18:53 #540 №689151 
>>689036
Такие в атмосфере сгорают, челябинский и то 11к весил, если ты имеешь ввиду около 300-400к тонн это всего 50 метров около ,это мощность бомбы на хиросиму которая упала, да и вероятней упадет где нить в океан.
Вот я все надеюсь что Апофис ебнит в 300 метров и 27 лярдов тонн, это удар будет 1000 мегатонн в 20 раз мощней царь бомбы
Аноним 02/11/21 Втр 11:04:24 #541 №689163 
>>689041
Ты и сам знаешь, что твоей маменьки.
Аноним 02/11/21 Втр 13:54:15 #542 №689216 
>>689151
Царь-бонба это хуйня мелкая. Теллер всерьёз двигал проект 10-гигатонной ёбы, и называл её backyard bomb. По очевидной причине - доставить её было невозможно ничем (она сама весила бы под пару тысяч тонн), а взрывать предполагалось у себя на территории в рамках угрозы взаимно гарантированного уничтожения, причём чтобы противнику (и всему остальному шарику) было ещё хуёвей, ибо он сдохнет не сразу, если конечно не взорвёт такой же девайс. Вот такой суицидальный шантаж уровня холодной войны. По счастью, увидев эту дичь, Теллера обозвали ёбаным шизоидом, каковым он собственно и был, а Эйзенхауэр ограничил мощность испытаний и принимаемого оружия 60 мегатоннами. После того как взорвали царь-бонбу, Теллер опять возбудился и стал двигать этот проект, но к тому времени подоспел запрет на испытания.
Аноним 02/11/21 Втр 14:00:13 #543 №689221 
>>689216
>По очевидной причине - доставить её было невозможно ничем (она сама весила бы под пару тысяч тонн)
А как же корабли?
Аноним 02/11/21 Втр 14:05:51 #544 №689222 
>>689221
Наверно можно было кораблём. Но по большому счёту на это было похуй, такой девайс практически гарантированно уничтожил бы всё человечество к хуям, или свёл бы его к минимуму.

Метеориты, впрочем, и побольше были в истории Земли. Чиксулуб, убивший динозавров, ёбнул на 100 тератонн - в 10к раз сильней.
Аноним 02/11/21 Втр 14:13:37 #545 №689229 
>>689222
10 гигатонн-то? Да не уничтожил бы он человечество, ты чего. Тамбора с такой мощностью извергся в 1815 году, сделал годовое похолодание лишь.
Аноним 02/11/21 Втр 14:25:09 #546 №689233 
>>689229
Ты не сравнивай вулкан и термоядерный девайс, который специально под эту цель заточен.
Аноним 02/11/21 Втр 14:29:25 #547 №689234 
>>689233
А че бы и не сравнить. Бонба более эффективно нанесет разрушения в окрестностях, но в целом по планете удар сопоставимый, просто на джоули смотришь и все.
Хотя ты прав, бомба меньше говна в атмосферу насрет, может эффект для планеты меньше будет.
Аноним 02/11/21 Втр 14:42:16 #548 №689237 
>>689234
>бомба меньше говна в атмосферу насрет
Кулстори уровня чистого ЯО.
crawl out through the fallout baby
cause they said this bomb was clean
Аноним 02/11/21 Втр 14:45:20 #549 №689238 
>>689237
Я не про изотопы щас, но да, чище чем взрыв реактора.
Сажа, зола, туф, прочая хуйня-малафья кубическими километрами высирается из вулкана. ЯО же даст в атмосферу лишь ядерный гриб. Ну может еще леса подожжет в окрестностях, но вряд ли заметно много.
Аноним 02/11/21 Втр 15:11:00 #550 №689249 
>>689238
>ЯО же даст в атмосферу лишь ядерный гриб.
Ага, потому что оно обычной мощности, причем ядерный гриб как раз и состоит из говн вперемешку с изотопами. Чего вдруг правильно спроектированному взрыву энергией с один из самых больших вулканов не дать точно такой же выброс на кубокилометры? Даже больше, учитывая концентрацию и его заточенность под это. И это только одно устройство. Такая хуйня засрёт всю земляшку, а несколько таких точно дадут массовое вымирание.
>но да, чище чем взрыв реактора
Чище чем взрыв реактора такой же энергетичности.
Аноним 02/11/21 Втр 15:18:50 #551 №689252 
15495450211630.jpg
>>689249
Как скажешь. Определенно будет НЕУДОБНО, но человечество не вымрет.
Аноним 02/11/21 Втр 15:27:43 #552 №689255 
>>689249
Хуя, кобальтовый гейзер Пелевина реально был в проекте
Аноним 02/11/21 Втр 18:37:59 #553 №689280 
Может ли так случится, что Земля случайно влетит в космические лучи из далеких объектов - и полностью стерилизуется. Каковы шансы вообще на это?
И как это ощутят живые существа?
Аноним 02/11/21 Втр 19:01:00 #554 №689291 
>>689280
Если полностью стерилизуется, то уже никак не оценят, очевидно же. А вообще вот, например https://ru.wikipedia.org/wiki/Ордовикско-силурийское_вымирание#Гипотеза_вспышки_гамма-излучения
Аноним 02/11/21 Втр 19:06:31 #555 №689293 
>>689291
А какие шансы влететь в такую вспышку?
Аноним 02/11/21 Втр 19:29:32 #556 №689295 
>>689293
50/50
Аноним 02/11/21 Втр 21:32:32 #557 №689319 
Если с помощью солнечного паруса можно разогнаться, то можно и затормозить?

Оцените план-капкан по запуску зонда к другой звезде:
1. Разгоняемся у солнца.
2. Тормозим у другой звёзды.

Но затея рискованная. Нужно, чтобы автоматика сработала идеально после нескольких сотен лет полета и зонд автоматически произвёл навигацию в другой звёздной системе.
Аноним 02/11/21 Втр 21:39:33 #558 №689320 
>>689249
А если сделать кумулятивный снаряд на 10гигатон, он сможет пронзить небеса литосферу?
Аноним 02/11/21 Втр 22:25:16 #559 №689324 
>>689319
Можно, но не эффективно.
Для торможения лучше всего годится магнитный парус.
Аноним 02/11/21 Втр 22:28:29 #560 №689325 
>>689295
Вспомнил анекдот про блондинку и динозавров
Аноним 02/11/21 Втр 22:32:06 #561 №689327 
>>689319
Что ты будешь делать со своим солнечным парусом если его продырявят микрочастицы?
Космос не стерилен
Аноним 02/11/21 Втр 23:13:31 #562 №689330 
>>689327
Сверну на время полета, компенсирую неравномерную тягу расположением паруса
Аноним 03/11/21 Срд 07:02:12 #563 №689367 
IMG0462.JPG
>>685225 (OP)PP>>685225 (OP)
Здрасте! Возник тупой вопрос, поэтому пишу сюда. Есть фотография, а на ней в правом нижнем углу что то похожее на планету. Если это и правда так, не мог бы кто то сказать или предположить какая именно?
Аноним 03/11/21 Срд 07:43:18 #564 №689371 
16359121333670.jpg
>>689367
Похоже на Йобу космических масштабов.
Аноним 03/11/21 Срд 08:53:58 #565 №689374 
>>689367
Это больше на блик похоже. Какой части неба фото, откуда, в какое время?
Аноним 03/11/21 Срд 09:31:24 #566 №689379 
>>689367
Открываешь в браузере Стеллариум указываешь местоположение, дату, время и находишь искомое.
Аноним 03/11/21 Срд 10:18:46 #567 №689389 
Если лунное затмение, это когда Луна прячется в тени Земли, то почему мы её видим?
Аноним 03/11/21 Срд 10:33:40 #568 №689391 
image.png
>>689389
Ночью темно, даже освещенная закатными преломленными лучами луна хорошо видна.
Аноним 03/11/21 Срд 11:16:12 #569 №689394 
>>689280
За все 4 млрд лет существования жизни на Земле не просиходило гамма-всплесков вблизи, значит, шансы не очень велики. Гамма-всплески происходят на космологических расстояниях, так что для нас никакой угрозы нет. Но если ебнет поблизости, то все, пиздец.
Аноним 03/11/21 Срд 11:28:07 #570 №689395 
>>689394
>а все 4 млрд лет существования жизни на Земле не просиходило гамма-всплесков вблизи
С чего ты взял?
Как минимум одно вымирание на вспышку приписывают.
Аноним 03/11/21 Срд 12:32:42 #571 №689397 
>>689391
Ааа, преломление, вот почему она красненькая...
Аноним 03/11/21 Срд 13:39:32 #572 №689404 
>>689395
Мало ли что там приписывают. Хуй к пизде тоже можно приписать, даже если его там нет. Если бы реальный гамма-всплеск ебанул неподалеку от Земли, мы бы сейчас тут не сидели.
Аноним 03/11/21 Срд 13:54:38 #573 №689407 
>>689404
Очень содержательное обоснование.
Аноним 03/11/21 Срд 14:03:14 #574 №689409 
>>689404
Если у тебя есть более логичное обоснование почему во время ордовикско-силурийского вымирания выпилены мелководные организмы по всей планете, а глубоководным норм - пиши. ГРБ отлично объясняет его.
Аноним 03/11/21 Срд 14:48:47 #575 №689424 
>>689407
>пук
>>689409
Какие есть пруфы, что это был гамма-всплеск? "Хорошо объясняет" - это говно собачье, хорошо объяснить можно что угодно чем угодно. Где реальные доказательства? Ты вообще представляешь себе, что такое гамма-всплеск? Их с космологических расстояний видят. Это чудовищный масштаб энерговыделения. Если бы такая хуйня пизданула около Земли, не спасся бы никто, никакие глубоководные организмы. Так что свое "хорошо объясняет" можешь засунуть в жопу тем, кто это "хорошее объяснение" высрал.
Аноним 03/11/21 Срд 14:51:52 #576 №689425 
>>689424
Значит нет объяснения, только собачий лай. Съеби обратно в /b/ или /po/ или откуда ты высрался и сри там дальше, и не возвращайся пока не научишься в рациональные аргументы.
Аноним 03/11/21 Срд 14:54:31 #577 №689427 
>>689424
>не спасся бы никто
Дегенерат пиздлявый. Выброс длится несколько секунд, половина земного шара узнала бы о нём только по косвенным признакам. Нассал тебе в глаз.
Аноним 03/11/21 Срд 15:27:26 #578 №689432 
Не было никаких случайных гамма всплески. Учите матчасть развитых цивилизаций и понятие "засева" планет земных групп.
Все это - целенаправленное отсеивание.
Аноним 03/11/21 Срд 17:07:18 #579 №689458 
>>689427
И да, чушок, за эти несколько секунд происходит такое колоссальное энерговыделение, что этого времени с лихвой хватило бы для стерилизации планеты. Ты даун, который не понимает, о чем пукает.
Аноним 03/11/21 Срд 18:41:17 #580 №689482 
>>689371
>>689374
Навряд ли. Тоже было такое предположение, но на других снимках "пятна" нет, также есть ещё пара фотографий той же части неба, где объект видно.
Аноним 03/11/21 Срд 18:43:06 #581 №689483 
>>689371
Годно. Извиняюсь что слегка не в тему, но что ты на снимке сделал? Выглядит как хорошо прорендеренная картинка, хз.
Аноним 03/11/21 Срд 19:03:47 #582 №689486 
>>689432
>матчасть развитых цивилизаций
Вот как после этого воспринимать серьезно этого серуна.
Зачем из зогача вылезаешь? Сиди с ебиномышленниками, тут тебе не рады.
Аноним 03/11/21 Срд 19:05:33 #583 №689487 
image.png
>>689483
В Фотошопе диалог «Уровни» для выделенной области, в нём кнопка «Авто», ну и руками ползунки повозить для лучшего результата. По сути оно берёт набор значений яркости точек в области (обычно он довольно узок, это будет видно на гистограмме) и изменяет этот набор так, чтобы сделать его максимально широким. Это приводит к тому, что слабо различающиеся исходные значения яркости точек становятся более заметными.
Аноним 03/11/21 Срд 21:27:41 #584 №689517 
>>6894С>>689487
Благодарю!
Аноним 03/11/21 Срд 23:45:49 #585 №689539 
>>687181
>железякой
Пздц. Погугли Состав Земли.
Аноним 04/11/21 Чтв 00:16:54 #586 №689542 
>>689539
В коре земли меньше железа чем в глубинах психеи. Что не так?
Аноним 04/11/21 Чтв 00:50:39 #587 №689549 
>>689542
Какая разница, если его хватит на тысячи звездолетов.
Аноним 04/11/21 Чтв 01:27:08 #588 №689554 
Максимальная высота для высокоэллиптической орбиты вокруг земли?
Аноним 04/11/21 Чтв 14:18:35 #589 №689649 
Кто-то может объяснить почему электроны из какого нибудь астероида за несколько миллионов световых лет абсолютно идентичны электронам входящим в состав моего ануса?
Аноним 04/11/21 Чтв 14:28:36 #590 №689659 
59c9.jpg
Космонавт Кудь-Сверчков изменил Родине, оказался действующим офицером НАТО и сбежал в Индиану чтобы жениться и наделать детишек с астронавткой и феминисткой Ауньон-Чанселлор.
Вопрос.
Какая фамилия будет у их детей? Очевидно же, что двойные фамилии это костыль, это путь в тупик, это купирование боли вместо лечения. Почему борьба за женские права свернула куда-то не туда?
Аноним 04/11/21 Чтв 15:00:00 #591 №689677 
>>689676 (OP)
>>689676 (OP)
>>689676 (OP)

ПЕРЕКОК
Аноним 04/11/21 Чтв 15:23:15 #592 №689687 
>>689659
Куньон-Сверчан.
Аноним 04/11/21 Чтв 20:13:41 #593 №689808 
>>689649
Неудобный для современной науки вопрос.
Аноним 04/11/21 Чтв 21:33:46 #594 №689821 
>>689808
Потому что ПОЧЕМУ БЫ И НЕТ
Аноним 04/11/21 Чтв 22:18:49 #595 №689830 
>>689808
В перекаченом треде два ответа, тупень.
Аноним 07/11/21 Вск 01:50:35 #596 №690334 
>>689830
И что, кретин? Пусть там хоть сто ответов экспертов с двача.
Аноним 08/11/21 Пнд 18:10:16 #597 №690665 
Слуште, а возможно создать полную компьютерную симуляцию космоса? Многие, или даже все нюансы Солнца например мы знаем, всякие нейтрино да радиация. Тоже самое с Землёй и Луной. Так что по идее можно ведь прописать все так скажем значения?.... Это было бы очень полезно, например чтоб проверить полетит ли ракета, или типа того... Понимаю что это очень муторно, но чисто в теории такое возможно провернуть? (фраза в теории возможно всё не принимается)
Аноним 10/11/21 Срд 05:26:33 #598 №690938 
>>690334
То, что тебе высраться своим охуительно важным мнением было бы логичнее там, а не здесь, где на тебя всем (кроме меня) похуй. Ну или не вякать вообще, если эксперты с двачей тебе не интересны.
Аноним 10/11/21 Срд 13:40:06 #599 №690957 
Почему недра планет не остывают?
Аноним 11/11/21 Чтв 22:22:08 #600 №691337 
>>690957 Из за термоядерных процессов, которые там происходят. Ну, насколько я знаю. Да и кстати со временем остывают, собственно как и звёзды.
Аноним 18/02/22 Птн 10:52:17 #601 №706816 
>>686700
>твои действия?
Мутирую в гидралиска.
Аноним 02/05/22 Пнд 12:41:11 #602 №712640 
>>689036
>Ваши действия?
Мутирую в гидралиска.
comments powered by Disqus

Отзывы и предложения